Silvestri501-600

  • Uploaded by: Linda Kuglarz
  • 0
  • 0
  • October 2019
  • PDF

This document was uploaded by user and they confirmed that they have the permission to share it. If you are author or own the copyright of this book, please report to us by using this DMCA report form. Report DMCA


Overview

Download & View Silvestri501-600 as PDF for free.

More details

  • Words: 22,646
  • Pages: 55
PN~Comp~Review~CD~501-600~1

PN Comprehensive Review CD Questions 501-600

{COMP: No Equations/Formulas; No questions}

501. A nurse finds a client lying tense in bed and staring at the cardiac monitor. The client states, “There sure are a lot of wires around there. I sure hope we don’t get hit by lightning!” The appropriate nursing response would be: 1. “Would you like a mild sedative to help you relax?” 2. “Oh, don’t worry; the weather is supposed to be sunny and clear today!” 3. “Yes, all those wires must be a little scary. Did someone explain what the cardiac monitor was for?” 4. “Your family can stay tonight if they wish.” Answer: 3 Rationale: The nurse should initially respond to validate the client’s concern and then should determine the client’s knowledge level of the cardiac monitor. This gives the nurse an opportunity to perform client education if necessary. Bringing in the family, friends, or a chaplain as an alternate resource may provide the client with additional psychological support. Pharmacological interventions should be considered only if necessary. Option 2 is a communication block. Test-Taking Strategy: Use the process of elimination. Because the client is the first priority, the focus of the nurse’s concern should be the client’s feelings, preferences, and choices. Option 3 validates the client’s anxiety and goes a step further to determine why the client might be anxious. Remember, address the client’s feelings first. Review therapeutic communication techniques if you had difficulty with this question. Level of Cognitive Ability: Application Client Needs: Physiological Integrity Integrated Process: Communication and Documentation Content Area: Adult Health/Cardiovascular Reference: Potter, P., & Perry, A. (2005). Fundamentals of nursing (6th ed.). St. Louis: Mosby, p. 437. 502. A maternity nurse inspects the amniotic fluid from a client following an amniotomy. The nurse documents that the amniotic fluid is normal if which of the following is noted? 1. It is light green with no odor 2. It is thick and white with no odor 3. It is clear and dark amber 4. It is pale with flecks of vernix Answer: 4 Rationale: Amniotic fluid is normally pale and straw-colored and may contain flecks of

PN~Comp~Review~CD~501-600~2

vernix caseosa. It should have a watery, not thick, consistency and no odor. Amber fluid suggests the presence of bilirubin, whereas greenish fluid may indicate the presence of meconium and suggests fetal distress. Test-Taking Strategy: Focus on the issue—that the amniotic fluid is normal. Noting the words “flecks of vernix” in option 4 will direct you to this option. Review this content if you had difficulty with this question. Level of Cognitive Ability: Comprehension Client Needs: Physiological Integrity Integrated Process: Nursing Process/Data Collection Content Area: Maternity/Intrapartum Reference: Leifer, G. (2005). Maternity nursing (9th ed.). Philadelphia: W.B. Saunders, p. 108. 503. A mental health nurse is caring for a client with a diagnosis of mania. The nurse selects which appropriate activity for this client? 1. Letter writing 2. Walking 3. Participating in a paint-by-number activity 4. Listening to music Answer: 2 Rationale: A person who is experiencing mania is overactive and full of energy, lacks concentration, and has poor impulse control. The client needs an activity that will facilitate use of excess energy, yet not endanger others during the process. Options 1, 3, and 4 are relatively sedate activities. Additionally, options 1 and 3 require concentration, a quality that is lacking in the manic state. Such activities may lead to increased frustration and anxiety for the client. Walking is an exercise that uses the large muscle groups of the body and is a great way to expend the increased energy that this client is experiencing. Test-Taking Strategy: Use the process of elimination, noting that options 1, 3, and 4 are similar in that they are relatively sedate activities. Review care to the client with mania if you had difficulty with this question. Level of Cognitive Ability: Application Client Needs: Psychosocial Integrity Integrated Process: Nursing Process/Implementation Content Area: Mental Health Reference: Stuart, G., & Laraia, M. (2005). Principles & practice of psychiatric nursing (8th ed.). St. Louis: Mosby, p. 355. 504. A nurse is assisting in preparing a client for a blood transfusion. Which of the following missing items would be most important for the nurse to retrieve or obtain in order to proceed with client identification? 1. Identification (ID) bracelet 2. Social security number 3. Client address 4. Medical record number Answer: 1

PN~Comp~Review~CD~501-600~3

Rationale: The identification bracelet that is placed on the client at admission is the most important item for proper client identification. The ID bracelet contains the client’s name, age, physician name, room number, and a hospital and/or medical record identification number. It contains the same information that is present on the client’s addressograph card, which is stamped on all client requisitions. The social security number and address are irrelevant for the procedure described. The medical record number would most likely be present on the client’s ID bracelet. Test-Taking Strategy: Use the process of elimination and focus on the issue—client identification. Begin to answer the question by eliminating options 2 and 3 first as incorrect. Choose option 1 over option 4 because it is the umbrella (global) option. Review the procedure for administering a blood transfusion if you had difficulty with this question. Level of Cognitive Ability: Application Client Needs: Safe, Effective Care Environment Integrated Process: Nursing Process/Implementation Content Area: Fundamental Skills Reference: Christensen, B., & Kockrow, E. (2003). Foundations of nursing (4th ed.). St. Louis: Mosby, p. 449. 505. A client requiring upcoming surgery is extremely anxious about the need for possible blood transfusion during or after surgery and is concerned about the risk of infection from contaminated blood. The nurse suggests that the client do which of the following as the most effective way to eliminate this risk? 1. Take iron supplements before surgery to boost hemoglobin levels 2. Request that any donated blood be screened twice by the blood bank 3. Ask a friend or family member to donate blood ahead of time 4. Donate autologous blood before the surgery Answer: 4 Rationale: Donating autologous blood to be reinfused as needed during or after surgery eliminates the risk of cross-infection from contaminated blood. The next most effective way to reduce the risk of contamination is to ask a family member to donate blood before surgery. Blood banks do not provide extra screening on request. Preoperative iron supplements are helpful for iron deficiency anemia, but are not most helpful in replacing blood lost during the surgery. Test-Taking Strategy: Use the process of elimination. Note the key words most effective. This tells you that more than one or all of the options may be partially or totally correct. Noting the key word autologous in option 4 will direct you to this option. Review disease transmission and blood donation procedures if you had difficulty with this question. Level of Cognitive Ability: Application Client Needs: Safe, Effective Care Environment Integrated Process: Nursing Process/Implementation Content Area: Fundamental Skills Reference: Christensen, B., & Kockrow, E. (2003). Foundations of nursing (4th ed.). St. Louis: Mosby, pp. 448-449.

PN~Comp~Review~CD~501-600~4

506. A nurse is collecting initial data on a newborn in the delivery room. Which of the following would the nurse expect to note when examining the umbilical cord of the newborn? 1. Two arteries and one vein 2. Two veins and one artery 3. One artery and one vein 4. Two arteries and two veins Answer: 1 Rationale: The umbilical cord is made up of two arteries to carry blood from the embryo to the chorionic villi and one vein that returns blood to the embryo. Test-Taking Strategy: Focus on the issue—a normal umbilical cord. Recall the anatomy of the umbilical cord to help you answer the question. Review this anatomy if you had difficulty with this question. Level of Cognitive Ability: Comprehension Client Needs: Physiological Integrity Integrated Process: Nursing Process/Data Collection Content Area: Maternity/Intrapartum Reference: McKinney, E., James, S., Murray, S., & Ashwill, J. (2005). Maternal-child nursing (2nd ed.). St. Louis: Elsevier, p. 522. 507. A pregnant client is seen in the health care clinic and is complaining of morning sickness. The client asks the nurse about measures to relieve this sick feeling. The appropriate suggestion to the client is: 1. Eat a high-carbohydrate diet 2. Eat a high-fat diet 3. Eat dry crackers before arising 4. Increase fluids with meals Answer: 3 Rationale: Some strategies for decreasing morning sickness are keeping crackers, Melba toast, or dry cereal at the bedside to eat before getting up in the morning; eating smaller, more frequent meals; decreasing fat intake; and consuming adequate fluid between meals. Options 1, 2, and 4 are not appropriate measures to relieve morning sickness. Test-Taking Strategy: Use the process of elimination. Note the relationship between the client’s complaint, morning sickness, and the correct option, to eat dry crackers before arising. Review the measures to relieve morning sickness if you had difficulty with this question. Level of Cognitive Ability: Application Client Needs: Health Promotion and Maintenance Integrated Process: Teaching/Learning Content Area: Maternity/Antepartum Reference: McKinney, E., James, S., Murray, S., & Ashwill, J. (2005). Maternal-child nursing (2nd ed.). St. Louis: Elsevier, p. 275. 508. A nurse is asked to check a unit of blood with another nurse before initiating a transfusion. The nurse notes that the blood type, Rh, expiration date, and unit number on the bag match the requisition. There is a discrepancy, however, in the client’s name. Which of the following actions would the nurse take?

PN~Comp~Review~CD~501-600~5

1. Hang the unit of blood because the blood information matches 2. Cross out the incorrect name and write in the correct one 3. Notify the physician that the client will not receive any blood 4. Call the blood bank about the discrepancy Answer: 4 Rationale: The nurse should call the blood bank to notify them of the discrepancy. The unit should not be hung, and information on the requisition or bag should not be altered in any way. The nurse assigned to the client may choose to call the physician, but the nature of that communication would be to report a delay in the transfusion due to the problem, not to report that there would be no transfusion. Test-Taking Strategy: The key word in the question is discrepancy. This tells you that the question is seeking a response that indicates the behavior of the nurse after analyzing this clinical problem. Knowledge of basic transfusion-related procedures should help you to eliminate each of the incorrect options. Review these procedures if you had difficulty with this question. Level of Cognitive Ability: Application Client Needs: Safe, Effective Care Environment Integrated Process: Nursing Process/Implementation Content Area: Fundamental Skills Reference: Christensen, B., & Kockrow, E. (2003). Foundations of nursing (4th ed.). St. Louis: Mosby, p. 449. 509. A pregnant client in the prenatal clinic states that her last menstrual period (LMP) began April 5 and ended April 12. According to Nägele’s rule, what would be the estimated date of delivery (EDD)? 1. January 21 2. December 19 3. January 12 4. January 19 Answer: 3 Rationale: Nägele’s rule is a noninvasive method of calculating the EDD as follows: add 7 days to the first day of the LMP, subtract 3 months, and add 1 year. This is based on the assumption that the menstrual cycle is 28 days. April 5 plus 7 days minus 3 months is January 12. Test-Taking Strategy: Use the process of elimination. The issue relates to the application of Nägele’s rule. Remember to use the first day of the LMP to calculate the EDD. Options 1, 2, and 4 are miscalculations. Review Nägele’s rule if you had difficulty with this question. Level of Cognitive Ability: Comprehension Client Needs: Health Promotion and Maintenance Integrated Process: Nursing Process/Data Collection Content Area: Maternity/Antepartum Reference: Leifer, G. (2005). Maternity nursing (9th ed.). Philadelphia: W.B. Saunders, p. 34. 510. A nurse in the prenatal clinic is collecting data on a client who is in the second

PN~Comp~Review~CD~501-600~6

trimester of pregnancy. The nurse notes that the pulse rate of the client has increased since the last visit. The nurse interprets this finding as most likely: 1. An indication of a cardiac problem 2. A sign of the client’s excitement about pregnancy 3. An abnormal finding 4. A normal finding Answer: 4 Rationale: Between 14 and 20 weeks, the pulse rate increases slowly, up 10 to 15 beats per minute, which lasts until term. Cardiac output and blood volume increase. Blood pressure decreases during the first half of pregnancy, returning to baseline in the second half of pregnancy. Options 1 and 3 are incorrect. Although excitement may cause an increase in pulse rate, the most likely cause is the combination of normal physiological changes that occur during pregnancy. Test-Taking Strategy: Use the process of elimination. Eliminate options 1 and 3 first because they are similar. Noting the key words most likely will easily direct you to option 4 from the remaining options. Review the physiological changes that occur in pregnancy if you had difficulty with this question. Level of Cognitive Ability: Comprehension Client Needs: Physiological Integrity Integrated Process: Nursing Process/Evaluation Content Area: Maternity/Antepartum Reference: Leifer, G. (2005). Maternity nursing (9th ed.). Philadelphia: W.B. Saunders, p. 39. 511. A nurse is reviewing the laboratory results of a pregnant client and notes that the hemoglobin level is decreased. Physiological anemia is documented in the client’s record by the physician. The nurse plans care knowing that this type of anemia is a result of: 1. Increased blood volume of the mother during pregnancy 2. Decreased metabolism of iron during pregnancy 3. Poor intake of iron-rich foods 4. Decreased maternal hemoglobin formation Answer: 1 Rationale: During the latter part of the first trimester, the blood volume of the mother increases more rapidly than blood cell production, leading to a decrease in the concentration of hemoglobin and erythrocytes. This is a normal process that causes a physiological anemia of pregnancy, or hemodilution. There is an increased metabolism of iron and maternal hemoglobin formation. The poor intake of iron-rich foods may cause an anemic condition, but physiological anemia of pregnancy occurs as a result of increased blood volume of the mother during pregnancy. Test-Taking Strategy: Use the process of elimination. Recalling the physiological changes contributing to anemia in pregnancy will direct you to option 1. Remember, during the latter part of the first trimester, the blood volume of the mother increases more rapidly than blood cell production, leading to a decrease in the concentration of hemoglobin and erythrocytes. Review these changes if you had difficulty with this question. Level of Cognitive Ability: Comprehension

PN~Comp~Review~CD~501-600~7

Client Needs: Physiological Integrity Integrated Process: Nursing Process/Planning Content Area: Maternity/Antepartum Reference: Leifer, G. (2005). Maternity nursing (9th ed.). Philadelphia: W.B. Saunders, p. 225. 512. A nurse is monitoring a client who is receiving a unit of packed red blood cells. Within an hour after the initiation of a transfusion, the nurse finds the client to be restless, with complaints of chills and back pain. The nurse notes that there is dark-colored urine in the Foley catheter drainage bag. The nurse interprets that the client is most likely experiencing which of the following reactions? 1. Delayed hemolytic 2. Acute hemolytic 3. Hyperkalemic 4. Allergic Answer: 2 Rationale: The client is experiencing an acute hemolytic reaction to the transfusion. The nurse in this instance would immediately stop the infusion and notify the registered nurse who will then notify the physician. A delayed hemolytic reaction typically occurs from 2 to 14 days after transfusion. A hyperkalemic reaction occurs when blood is transfused that has been stored for too long, resulting in red blood cell hemolysis. The client experiencing a hyperkalemic type of reaction would exhibit nausea, muscle weakness or paresthesias, apprehension, bradycardia, ECG changes, and possibly cardiac arrest. An allergic reaction is characterized by flushing, nausea and vomiting, respiratory stridor, hypotension, and other signs of anaphylaxis. Test-Taking Strategy: Focus on the data in the question. Noting the phrase “within an hour after the initiation of a transfusion” will direct you to option 2. Review these types of reactions if you had difficulty with this question. Level of Cognitive Ability: Analysis Client Needs: Physiological Integrity Integrated Process: Nursing Process/Evaluation Content Area: Fundamental Skills References: Christensen, B., & Kockrow, E. (2003). Foundations of nursing (4th ed.). St. Louis: Mosby, p. 450. deWit, S. (2005). Fundamental concepts and skills for nursing. Philadelphia: W.B. Saunders, p. 723. Linton, A., & Maebius, N. (2003). Introduction to medical-surgical nursing (3rd ed.). Philadelphia: W.B. Saunders, p. 521. 513. A client receiving a blood transfusion begins to exhibit flushing, stridor, and a drop in blood pressure. The nurse would initially obtain which of the following medications from the emergency cart to have ready for use as ordered? 1. Aminophylline (theophylline) 2. Lidocaine 3. Norepinephrine 4. Epinephrine (Adrenalin)

PN~Comp~Review~CD~501-600~8

Answer: 4 Rationale: The symptoms exhibited by the client are compatible with an allergic reaction to the transfusion. Other common symptoms of allergic reaction are nausea and vomiting, diarrhea, and loss of consciousness. The nurse prepares to administer epinephrine and corticosteroid medications as ordered. Norepinephrine is a sympathetic agonist used to treat hypotension, but is not indicated in an allergic reaction. Lidocaine is an antidysrhythmic medication. Aminophylline is a bronchodilator, which could possibly be prescribed if needed to treat bronchospasm. Test-Taking Strategy: Note the key word initially. This tells you that more than one or all of the options may be partially or totally correct. However, only one of the options is the initial action of the nurse. In this case, you would eliminate options 2 and 3 first. Select option 4 over option 1 because it is the first-line agent used in management of severe allergic (anaphylactic) reactions. Review the emergency treatment for a blood transfusion reaction if you had difficulty with this question. Level of Cognitive Ability: Application Client Needs: Physiological Integrity Integrated Process: Nursing Process/Planning Content Area: Pharmacology References: Linton, A., & Maebius, N. (2003). Introduction to medical-surgical nursing (3rd ed.). Philadelphia: W.B. Saunders, p. 521. McKenry, L., & Salerno, E. (2003). Mosby’s pharmacology in nursing (21st ed.). St. Louis: Mosby, pp. 462-463. 514. A nurse is reinforcing instructions to a client taking divalproex sodium (Depakote). The nurse tells the client to return to the clinic for follow-up laboratory studies of which of the following tests? 1. Liver function studies 2. Renal function studies 3. Glucose tolerance test 4. Electrolyte studies Answer: 1 Rationale: Divalproex sodium, an anticonvulsant, can cause hepatotoxicity, which is potentially fatal. The nurse instructs the client to return to the clinic for follow-up liver function studies, such as lactate dehydrogenase (LDH), serum glutamic-oxaloacetic transaminase (SGOT), serum glutamic-pyruvic transaminase (SGPT), and ammonia levels. This is especially indicated in the first 6 months of therapy. The laboratory studies identified in options 2, 3, and 4 are not specifically related to the administration of this medication. Test-Taking Strategy: To answer this question accurately, recall that this medication can lead to hepatotoxicity. This will direct you to option 1. Review this medication if you had difficulty with this question. Level of Cognitive Ability: Application Client Needs: Physiological Integrity Integrated Process: Nursing Process/Implementation Content Area: Adult Health/Neurological Reference: Hodgson, B., & Kizior, R. (2005). Saunders nursing drug handbook 2005.

PN~Comp~Review~CD~501-600~9

Philadelphia: W.B. Saunders, p. 1094. 515. A client is seen by the physician, and Bell’s palsy is suspected. Which of the following signs and symptoms would the nurse expect to note in the client if this disorder is present? 1. Tingling sensations of the eyelid 2. Burning pain in the nose with intermittent facial paralysis 3. Speech or chewing difficulties accompanied by facial droop 4. Stabbing facial pain with intermittent tingling sensations in the eyes Answer: 3 Rationale: Bell’s palsy is a one-sided facial paralysis from compression of the facial nerve (CN VII). There is facial droop from paralysis of the facial muscles, increased lacrimation, speech or chewing difficulties, and painful sensations in the eye, in the face, or behind the ear. Options 1, 2, and 4 are not characteristics of Bell’s palsy. Test-Taking Strategy: Use the process of elimination. Eliminate options 1 and 4 first because they are similar. Remember that a palsy is a type of paralysis. Knowing that the symptoms do not “come and go” (are not intermittent) helps you eliminate option 2 next. Review the signs and symptoms of Bell’s palsy if you had difficulty with this question. Level of Cognitive Ability: Analysis Client Needs: Physiological Integrity Integrated Process: Nursing Process/Data Collection Content Area: Adult Health/Neurological Reference: Christensen, B., & Kockrow, E. (2003). Adult health nursing (4th ed.). St. Louis: Mosby, p. 643. 516. A client sustains a burn injury to the entire right and left arms, right leg, and anterior thorax. According to the “rule of nines,” the nurse would determine that this injury constitutes which of the following body percentages? 1. 27% 2. 36% 3. 45% 4. 54% Answer: 4 Rationale: According to the “rule of nines,” the right arm is equal to 9% and the left arm is equal to 9%. The right leg is equal to 18% and the left leg is equal to 18%. The anterior thorax is equal to 18% and the posterior thorax is equal to 18%. The head is equal to 9% and the perineum is 1%. If the anterior thorax, the right leg, and the right and left arms were burned, the area of injury would equal 54% according to the “rule of nines.” Test-Taking Strategy: Knowledge of the percentages associated with the “rule of nines” is required to answer this question. From this point, you would be able to determine the percentage of burn injury and use the process of elimination to select the correct option. Review the rule of nines if you had difficulty with this question. Level of Cognitive Ability: Comprehension Client Needs: Physiological Integrity Integrated Process: Nursing Process/Data Collection

PN~Comp~Review~CD~501-600~10

Content Area: Adult Health/Integumentary Reference: Christensen, B., & Kockrow, E. (2003). Adult health nursing (4th ed.). St. Louis: Mosby, p. 90. 517. A nurse is collecting neurological data on a poststroke adult client. Which of the following techniques will the nurse perform to adequately check proprioception? 1. Firmly stroke the lateral sole of the foot and under the toes with a blunt instrument 2. Tap the Achilles tendon using the reflex hammer 3. Gently prick the client’s skin on the dorsum of the foot in two places 4. Hold the sides of the client’s great toe and, while moving it, ask the client to describe the toe’s position Answer: 4 Rationale: Proprioception is tested by holding the sides of the client’s great toe and, while moving it, asking the client to describe the toe’s position. The plantar reflex is elicited in option 1. Normally, the toes plantar-flex, but in an abnormal reflex, the toes dorsiflex and fan out. Option 2 identifies the assessment for gastrocnemius muscle contraction, and option 3 tests two-point discrimination. Test-Taking Strategy: Knowledge of general neurological data collection techniques is helpful in answering this question. Note the relationship between the term “proprioception” in the question and the word “position” in the correct option. Review general neurological assessment if you had difficulty with this question. Level of Cognitive Ability: Application Client Needs: Health Promotion and Maintenance Integrated Process: Nursing Process/Data Collection Content Area: Adult Health/Neurological Reference: Potter, P., & Perry, A. (2005). Fundamentals of nursing (6th ed.). St. Louis: Mosby, p. 933. 518. A nurse is providing instructions to a client with otitis media receiving amoxicillin (Amoxil) 500 mg orally every 8 hours. Which of the following statements by the client would indicate an understanding of the adverse effects related to the medication? 1. “A headache may mean that I need to discontinue the medication.” 2. “I may become dizzy from the medication.” 3. “Constipation means that the medication needs to be stopped.” 4. “If I get diarrhea, I need to call the doctor.” Answer: 4 Rationale: Amoxicillin is a penicillin. Adverse reactions include superinfections such as potentially fatal antibiotic-associated colitis, which results from altered bacterial balance. Symptoms include abdominal cramps, severe watery diarrhea, and fever. Frequent side effects include gastrointestinal (GI) disturbances, headache, and oral or vaginal candidiasis. Options 1, 2, and 3 are incorrect. Test-Taking Strategy: Note the key words understanding of the adverse effects. Remember, antibiotics have a tendency to produce GI side effects. If you can remember that this medication is classified as an antibiotic, you will select option 4 by the process of elimination. Review the adverse effects of this medication if you had difficulty with this question.

PN~Comp~Review~CD~501-600~11

Level of Cognitive Ability: Analysis Client Needs: Physiological Integrity Integrated Process: Nursing Process/Evaluation Content Area: Pharmacology Reference: Hodgson, B., & Kizior, R. (2005). Saunders nursing drug handbook 2005. Philadelphia: W. B. Saunders, p. 60. 519. A nurse is providing instructions to a client with glaucoma receiving acetazolamide (Diamox) daily. Which of the following statements by the client indicates an understanding of the adverse effects related to the medication? 1. “Constipation will probably be a problem with this medication.” 2. “If difficulty with swallowing occurs, I need to taper the medication.” 3. “I need to call the doctor if I get dark-colored urine and stools.” 4. “If I get really irritable, I need to stop the medication.” Answer: 3 Rationale: Acetazolamide is a carbonic anhydrase inhibitor. Nephrotoxicity and hepatotoxicity occur and are manifested by dark-colored urine and stools, pain in the lower back, jaundice, dysuria, crystalluria, renal colic, and calculi. Bone marrow depression also may occur. Options 1, 2, and 4 are incorrect client statements. Test-Taking Strategy: Use the process of elimination. Eliminate options 2 and 4 first, using general principles related to medication therapy. The client should not taper or stop medication without physician consultation. Noting the key words adverse effects in the question will assist in directing you to option 3. Additionally, remembering that acetazolamide is nephrotoxic and hepatotoxic will assist in directing you to the correct option. Review this medication if you had difficulty with this question. Level of Cognitive Ability: Analysis Client Needs: Physiological Integrity Integrated Process: Nursing Process/Evaluation Content Area: Pharmacology Reference: Hodgson, B., & Kizior, R. (2005). Saunders nursing drug handbook 2005. Philadelphia: W.B. Saunders, p. 10. 520. A nurse is caring for a client at home who has had a tracheostomy tube for several months. The nurse monitors the client for complications associated with the long-term tracheotomy and suspects tracheoesophageal fistula if which of the following is noted in the client? 1. Abdominal distention 2. Purulent drainage around the tracheotomy site 3. Excessive secretions from the tracheotomy site 4. Inability to pass a suction catheter through the tracheotomy Answer: 1 Rationale: Necrosis of the tracheal wall can lead to an artificial opening between the posterior trachea and the esophagus. This problem is called tracheoesophageal fistula. The fistula allows air to escape into the stomach, causing abdominal distention. It also can cause aspiration of gastric contents. Option 2 may indicate an infection. Option 3 may indicate the need for more frequent suctioning. Option 4 may indicate an

PN~Comp~Review~CD~501-600~12

obstruction of some sort or the presence of bronchoconstriction. Test-Taking Strategy: A fistula is an artificial opening. Remembering that tracheoesophageal indicates trachea to esophagus, review the options and use the process of elimination. If you think of air moving from the trachea to the esophagus, you will know that abdominal distention occurs with this condition. Review the signs of this complication if you had difficulty with this question. Level of Cognitive Ability: Analysis Client Needs: Physiological Integrity Integrated Process: Nursing Process/Data Collection Content Area: Adult Health/Respiratory Reference: Black, J., & Hawks, J. (2005). Medical-surgical nursing: Clinical management for positive outcomes (7th ed.). Philadelphia: W.B. Saunders, p. 1779. 521. A nurse is observing a student preparing to suction a pediatric client through a tracheostomy. The nurse intervenes if the student verbalizes to: 1. Use a twisting motion on the catheter when withdrawing the catheter 2. Apply continuous suction when withdrawing the catheter 3. Limit insertion and suctioning time to 5 seconds 4. Reoxygenate the child between suction catheter passes Answer: 2 Rationale: The nurse would apply intermittent (not continuous) suction on the catheter during withdrawal to prevent trauma to the mucosa. Options 1, 3, and 4 represent correct interventions regarding this procedure. Test Taking Strategy: Note the key word intervenes. This word indicates a false response question and that you need to select the incorrect student statement. Noting the word “continuous” in option 2 will direct you to this option. If you are unfamiliar with the procedure, review this content. Level of Cognitive Ability: Analysis Client Needs: Safe, Effective Care Environment Integrated Process: Nursing Process/Implementation Content Area: Leadership/Management Reference: Price, D., & Gwin, J. (2005). Thompson’s pediatric nursing (9th ed.). Philadelphia: W.B. Saunders, p. 374. 522. A nurse is preparing to administer an acetaminophen (Tylenol) suppository to a child. The nurse plans to: 1. Position the child on the right side with the left leg flexed 2. Ask the child to hold the breath during insertion of the suppository 3. Insert the suppository 1 to 2 cm into the rectum 4. Ask the child to expel the suppository after it has been inserted Answer: 3 Rationale: When administering a suppository to a child, the child should be positioned on the left side with the right leg flexed. The child should be asked to take a deep breath to further relax the sphincter. The suppository is gently inserted past the internal sphincter and the distance required to place the medication is approximately 1 to 2 cm. After insertion, the buttocks should be held together until the urge to expel the

PN~Comp~Review~CD~501-600~13

suppository has passed. Test-Taking Strategy: Focus on the procedure—the appropriate method for administering a rectal suppository to a child. Visualizing the procedure and noting that the question is addressing the pediatric client will direct you to option 3. If you are unfamiliar with this procedure, review this content. Level of Cognitive Ability: Application Client Needs: Physiological Integrity Integrated Process: Nursing Process/Planning Content Area: Child Health References: Leifer, G. (2003). Introduction to maternity & pediatric nursing (4th ed.). Philadelphia: W.B. Saunders, p. 514. McKinney, E., James, S., Murray, S., & Ashwill, J. (2005). Maternal-child nursing (2nd ed.). St. Louis: Elsevier, p. 980. 523. A nurse is reviewing the record of a child admitted to the hospital with nephrotic syndrome. Which finding would the nurse expect to note documented in the record? 1. Hyperalbuminemia 2. Weight loss 3. Increased appetite 4. Proteinuria Answer: 4 Rationale: The term nephrotic syndrome refers to a kidney disorder characterized by proteinuria, hypoalbuminemia, and edema. The child experiences fatigue, anorexia, increased weight, abdominal pain, and a normal blood pressure. Test Taking Strategy: Use the process of elimination. Recalling that this syndrome is related to the renal system will assist in directing you to option 4. If you are unfamiliar with the signs associated with this disorder, review this content. Level of Cognitive Ability: Comprehension Client Needs: Physiological Integrity Integrated Process: Nursing Process/Data Collection Content Area: Child Health Reference: Price, D., & Gwin, J. (2005). Thompson’s pediatric nursing (9th ed.). Philadelphia: W.B. Saunders, p. 247. 524. A nurse is reviewing the record of a child scheduled for a physician’s visit. Before data collection, the nurse notes documentation that the child has enuresis. Based on this diagnosis, the nurse plans to focus on which of the following when collecting data? 1. Bowel function 2. Bladder function 3. Nutritional status and weight gain 4. Motor development Answer: 2 Rationale: Enuresis refers to a condition in which the child is unable to control bladder function although he or she has reached an age at which control of voiding is expected. Nocturnal enuresis or bed-wetting is common in children. Test-Taking Strategy: If you are unfamiliar with the term enuresis, use medical

PN~Comp~Review~CD~501-600~14

terminology and break the word down into parts. Associate the word with the urinary system to assist in directing you to the correct option. Review this disorder if you had difficulty with this question. Level of Cognitive Ability: Comprehension Client Needs: Physiological Integrity Integrated Process: Nursing Process/Data Collection Content Area: Child Health Reference: Price, D., & Gwin, J. (2005). Thompson’s pediatric nursing (9th ed.). Philadelphia: W.B. Saunders, p. 217. 525. A nurse is preparing to administer an intramuscular injection to a 1-year-old child. Which of the following locations would the nurse select to administer the medication? 1. Ventrogluteal muscle 2. Deltoid muscle 3. Dorsogluteal muscle 4. Vastus lateralis muscle Answer: 4 Rationale: The vastus lateralis muscle is the best choice for all age groups and should always be used in children younger than 3 years of age. The ventrogluteal muscle is safe for children older than 18 months because it is free of major blood vessels and nerves. The dorsogluteal muscle develops with walking, so it should not be used until the child has been walking for at least 1 year. The deltoid muscle is not used for children because the small muscle mass cannot hold large volumes of medication or medications that must be injected deep into the muscle mass. Test Taking Strategy: Focus on the age of the child identified in the question to assist in directing you to the correct option. Remember, the vastus lateralis muscle is the best choice for all age groups and should always be used in children younger than 3 years of age. If you are unfamiliar with this procedure, review this content. Level of Cognitive Ability: Application Client Needs: Physiological Integrity Integrated Process: Nursing Process/Implementation Content Area: Child Health Reference: Price, D., & Gwin, J. (2005). Thompson’s pediatric nursing (9th ed.). Philadelphia: W.B. Saunders, pp. 366-367. 526. A nurse is preparing to administer an enteral feeding through a nasogastric tube. The nurse would place the client in which position during and after the feedings? 1. Supine 2. Fowler’s 3. Sims 4. Trendelenburg Answer: 2 Rationale: The client is placed with the head of the bed elevated 30 to 45 degrees both during and after feedings to prevent aspiration. The positions identified in options 1, 3, and 4 place the client at risk for aspiration. Test-Taking Strategy: Think about the risks associated with tube feedings. Recalling that

PN~Comp~Review~CD~501-600~15

the risk of aspiration exists will direct you to option 2. Review care to the client receiving tube feedings if you had difficulty with this question. Level of Cognitive Ability: Application Client Needs: Physiological Integrity Integrated Process: Nursing Process/Implementation Content Area: Adult Health/Gastrointestinal Reference: deWit, S. (2005). Fundamental concepts and skills for nursing. Philadelphia: W.B. Saunders, p. 481. 527. A client is being prepared for electroconvulsive therapy (ECT). The nurse’s plan of care for the night before ECT treatment includes ensuring that the client: 1. Is placed on “nothing by mouth” status for 18 to 24 hours 2. Receives no visitors and participates in limited unit activities 3. Shampoos and dries hair, freeing it of all hair spray and creams 4. Does not smoke cigarettes Answer: 3 Rationale: The client is instructed to shampoo and dry the hair the night before ECT treatment. In addition, the client is instructed not to use hair sprays or creams before ECT to reduce the risk of burns. While the client is NPO for 6 to 8 hours before treatment, the 18 to 24 hours identified in option 1 is not necessary. Some hospitals place inpatient clients on NPO status at midnight before ECT in the morning. Some clients who are on cardiovascular medication may be instructed to take their medicine with sips of water several hours before ECT. Option 2 is incorrect as is option 4, although smoking is always discouraged. In addition, informed consent, minimization of client and family anxieties and fears concerning the procedure, and keeping the client free from injury are treatment goals for the client undergoing ECT. Test-Taking Strategy: Focus on knowledge of the pretreatment preparation for ECT to answer the question and use the process of elimination. Remember, the client is instructed to shampoo and dry the hair the night before ECT treatment to reduce the risk of burns. Review the preprocedure preparation for ECT if you had difficulty with this question. Level of Cognitive Ability: Application Client Needs: Physiological Integrity Integrated Process: Nursing Process/Planning Content Area: Mental Health Reference: Stuart, G., & Laraia, M. (2005). Principles & practice of psychiatric nursing (8th ed.). St. Louis: Mosby, p. 606. 528. A nurse is assisting a physician with insertion of an endotracheal tube (ETT). The nurse should plan to assure that which of the following is done as a final measure to determine correct tube placement? 1. Hyperoxygenate the client 2. Listen for bilateral breath sounds 3. Tape the tube securely in place 4. Verify placement by chest x-ray Answer: 4 Rationale: The final measure to determine ETT placement is to verify by chest x-ray.

PN~Comp~Review~CD~501-600~16

The chest x-ray shows the exact placement of the tube in the trachea, which should be above the bifurcation of the right and left mainstream bronchi. The other options are incorrect because they are completed initially after tube placement. Test-Taking Strategy: The key words in the question are final measure and correct tube placement. These words tell you that you are looking for a correct item, and also imply a time sequence. Knowing that the client is hyperoxygenated before and immediately after insertion, you would eliminate option 1. Option 2 is eliminated next because it is the initial means used to verify placement, not the final one. Option 3 is done before option 4 to avoid tube displacement before or during the x-ray. Review the procedures for verifying correct ETT placement if you had difficulty with this question. Level of Cognitive Ability: Application Client Needs: Physiological Integrity Integrated Process: Nursing Process/Implementation Content Area: Adult Health/Respiratory References: Black, J., & Hawks, J. (2005). Medical-surgical nursing: Clinical management for positive outcomes (7th ed.). Philadelphia: W.B. Saunders, p. 1883. deWit, S. (2005). Fundamental concepts and skills for nursing. Philadelphia: W.B. Saunders, p. 479. 529. An older client with ischemic heart disease has experienced an episode of dizziness and shortness of breath. The nurse reviews the plan of care and notes documentation of a nursing diagnosis of decreased cardiac output related to possible dysrhythmias as evidenced by dyspnea and a syncopal episode. The nurse plans to take which most important action in the care of the client? 1. Measure blood pressure every 4 hours 2. Monitor oxygen saturation levels 3. Check capillary refill at least once per shift 4. Place the client on a cardiac monitor Answer: 4 Rationale: The client with decreased cardiac output and possible dysrhythmias should be placed on continuous cardiac monitoring so myocardial perfusion can be most accurately assessed. Other cardiovascular assessments should be made at least every 2 hours initially. Test-Taking Strategy: Focus on the data provided in the question and note the key words possible dysrhythmias. These words along with the words “most important action” limit the choices to options 2 or 4. Since option 4 is a more direct measurement about cardiovascular status, it is chosen over option 2 (which in the absence of cardiac monitoring would be a reasonable choice). Review appropriate goals for the nursing diagnosis of decreased cardiac output if you had difficulty with this question. Level of Cognitive Ability: Analysis Client Needs: Physiological Integrity Integrated Process: Nursing Process/Planning Content Area: Adult Health/Cardiovascular References: Gulanick, M., Myers, J., Klopp, A., Gradishar, D., Galanes, S., & Puzas, M. (2003). Nursing care plans: Nursing diagnosis and intervention (5th ed.). St. Louis: Mosby, p. 30.

PN~Comp~Review~CD~501-600~17

Lewis, S., Heitkemper, M., & Dirksen, S. (2004). Medical-surgical nursing: Assessment and management of clinical problems (6th ed.). St. Louis: Mosby, p. 1770. 530. A nurse is planning adaptations needed for activities of daily living for the client with cardiac disease. The nurse would incorporate which of the following in discussions with the client? 1. Consume 1 to 2 ounces of liquor each night to promote vasodilation 2. Try to engage in vigorous activity to strengthen cardiac reserve 3. Consume adequate daily fiber to prevent straining and constipation 4. Increase fluid intake to 3000 mL per day to promote renal perfusion Answer: 3 Rationale: Standard instructions for a client with cardiac disease include, among others, lifestyle changes such as decreasing alcohol intake, avoiding activities that increase the demands on the heart, instituting a bowel regimen to prevent straining and constipation, and maintaining fluid and electrolyte balance. Increasing fluid intake to 3000 mL could lead to increased blood volume and an increased workload on the heart in the client with cardiac disease. Test-Taking Strategy: Use the process of elimination and knowledge regarding the effects of cardiac disease to answer the question. Focusing on the client’s diagnosis will assist in eliminating options 1 and 4. From the remaining options, noting the word “vigorous” in option 2 will assist in eliminating this option. Review care to the client with cardiac disease if you had difficulty with this question. Level of Cognitive Ability: Application Client Needs: Physiological Integrity Integrated Process: Nursing Process/Planning Content Area: Adult Health/Cardiovascular Reference: Phipps, W., Monahan, F., Sands, J., Marek, J., & Neighbors, M. (2003). Medical-surgical nursing: health and illness perspectives (7th ed.). St. Louis: Mosby, p. 725. 531. A nurse is assigned to care for a client with a diagnosis of hepatic encephalopathy. The nurse is preparing to administer medications to the client expecting that which medication is prescribed? 1. Magnesium hydroxide (Milk of Magnesia) 2. Phenolphthalein (Ex-Lax) 3. Psyllium hydrophilic mucilloid (Metamucil) 4. Lactulose syrup (Chronulac) Answer: 4 Rationale: Lactulose syrup is a hyperosmotic laxative agent that has the adjunct benefit of lowering serum ammonia levels. This occurs because the medication lowers bowel pH, which aids the conversion of ammonia in the colon to the ammonium ion, which is poorly absorbed. Magnesium hydroxide is a saline laxative, while phenolphthalein is a stimulant laxative. Psyllium hydrophilic mucilloid is a bulk laxative. Test-Taking Strategy: To answer this question accurately, it is necessary to know the pathophysiology related to hepatic encephalopathy, the specific effects of prescribed medications, and the benefits of use in the client with impaired liver function.

PN~Comp~Review~CD~501-600~18

Remember, lactulose syrup is a hyperosmotic laxative agent that has the adjunct benefit of lowering serum ammonia levels. Review care to the client with hepatic encephalopathy if you had difficulty with this question. Level of Cognitive Ability: Analysis Client Needs: Physiological Integrity Integrated Process: Nursing Process/Planning Content Area: Pharmacology Reference: Hodgson, B., & Kizior, R. (2005). Saunders nursing drug handbook 2005. Philadelphia: W.B. Saunders, p. 612. 532. A client has a history of seizures. The physician ordered amitriptyline (Elavil) 25 mg three times daily. The nurse plans care knowing that the client is at risk for injury because of which adverse effect of amitriptyline? 1. Decreased platelet aggregation 2. Decreased seizure threshold 3. Decreased mental acuity 4. Depressed immunological system Answer: 2 Rationale: Amitriptyline, a tricyclic antidepressant, lowers the seizure threshold, increasing the risk of seizures. This may not be the medication of choice for a client who is already at risk for seizure activity. Options 1, 3, and 4 are unrelated to the use of this medication. Test-Taking Strategy: Note that the client has a history of seizures. Also noting the relation of the client’s diagnosis and option 2 will direct you to this option. Review the adverse effects of this medication if you had difficulty with this question. Level of Cognitive Ability: Analysis Client Needs: Physiological Integrity Integrated Process: Nursing Process/Planning Content Area: Mental Health Reference: McKenry, L., & Salerno, E. (2003). Mosby’s pharmacology in nursing (21st ed.). St. Louis: Mosby, p. 418. 533. An adult client just admitted to the hospital with heart failure also has a history of diabetes mellitus. The nurse calls the physician to verify an order for which medication that the client was taking prior to admission? 1. NPH insulin 2. Chlorpropamide (Diabinese) 3. Regular insulin 4. Acarbose (Precose) Answer: 2 Rationale: Chlorpropamide is an oral hypoglycemic agent that exerts an antidiuretic effect and should be administered cautiously or avoided in the client with cardiac impairment or fluid retention. It is a first-generation sulfonylurea. Insulin does not cause or aggravate fluid retention. Acarbose is a miscellaneous oral hypoglycemic agent. Test-Taking Strategy: Use the process of elimination. Eliminate options 1 and 3 first because they are similar. From the remaining options, recall that chlorpropamide causes

PN~Comp~Review~CD~501-600~19

fluid retention. This will direct you to option 2. Review the adverse effects of chlorpropamide if you had difficulty with this question. Level of Cognitive Ability: Analysis Client Needs: Physiological Integrity Integrated Process: Nursing Process/Implementation Content Area: Adult Health/Endocrine Reference: Lehne, R. (2004). Pharmacology for nursing care (5th ed.). Philadelphia: W.B. Saunders, pp. 608, 617. 534. A nurse is assisting in teaching a series of classes on maintaining a healthy pregnancy. The goal for the class is: “The pregnant woman will verbalize measures that may prevent traumatic conditions distressing to the fetus.” Based on this goal, which of the following would be a part of the teaching plan for this class? 1. Travel precautions and use of shoulder seat belts 2. Use of over-the-counter medications 3. Fetotoxic substances in the workplace 4. Effects of secondary cigarette smoke on the fetus Answer: 1 Rationale: Placental separation as a result of uterine distortion can occur from trauma, such as in car accidents. Placental separation decreases or shuts off uteroplacental circulation. Partial separation will result in fetal distress, with the distress increasing depending on the degree of separation. Complete separation leads to sudden severe fetal distress followed by fetal death. Use of the shoulder seat belt decreases the risk of placental separation by preventing the traumatic flexion of the woman’s body from sharp braking or impact, if an accident occurs. Test-Taking Strategy: Note the issue of the question—traumatic conditions. Option 1 specifically addresses trauma from an injury such as a car or other vehicle accident. Options 2, 3, and 4 are also important teaching points but are not directly related to the issue of the question. Review measures to prevent trauma during pregnancy if you had difficulty with this question. Level of Cognitive Ability: Analysis Client Needs: Health Promotion and Maintenance Integrated Process: Nursing Process/Planning Content Area: Maternity/Antepartum Reference: Leifer, G. (2003). Introduction to maternity & pediatric nursing (4th ed.). Philadelphia: W.B. Saunders, pp. 57-58. 535. A client with Parkinson’s disease “freezes” while ambulating, increasing the risk for falls. Which suggestion will the nurse include in the client’s plan of care to alleviate this problem? 1. Stand erect and use a cane to ambulate 2. Keep the feet close together while ambulating and using a walker 3. Consciously think about walking over imaginary lines on the floor 4. Use a wheelchair to move around Answer: 3 Rationale: Clients with Parkinson’s disease can develop bradykinesia (slow movement)

PN~Comp~Review~CD~501-600~20

or akinesia (freezing or no movement). Having these individuals imagine lines on the floor to step over can keep them moving forward. While standing erect and using a cane can help prevent falls, these measures will not help a person with akinesia move forward. Clients with Parkinson’s disease should walk with a wide gait, not with the feet close together. A wheelchair should be used only when the client can no longer ambulate with assistive devices such as canes or walkers. Test-Taking Strategy: Focus on the issue—a plan for a client who “freezes” while ambulating. Visualize each option and note that option 3 encourages forward movement while ambulating. Review the special needs of the client with Parkinson’s disease if you had difficulty with this question. Level of Cognitive Ability: Application Client Needs: Safe, Effective Care Environment Integrated Process: Nursing Process/Planning Content Area: Adult Health/Neurological Reference: Christensen, B., & Kockrow, E. (2003). Adult health nursing (4th ed.). St. Louis: Mosby, p. 630. 536. A nurse is caring for an African-American client. The nurse enters the room, and following a greeting and introduction to the client, the nurse begins to describe the procedure for a prescribed soapsuds enema. The client turns away from the nurse. Which nursing action is appropriate? 1. Continue with the explanation 2. Tell the client that the enemas are necessary 3. Walk around to the client and ask the client what the problem might be 4. Leave the room and return later to continue with the explanation Answer: 1 Rationale: In the African-American culture, direct eye contact may be viewed as being rude. If the client turns away from the nurse during a conversation, the best action is to continue with the conversation. Walking around to the client so that the nurse faces the client is in direct conflict with the cultural practice. Leaving the room and returning later to continue with the explanation may be viewed as a rude gesture by the client. Option 2 is nontherapeutic. Test-Taking Strategy: Understanding the characteristics of this cultural group will assist in answering the question. Also, use of therapeutic communication techniques will direct you to option 1. Review the characteristics of this cultural group if you had difficulty with this question. Level of Cognitive Ability: Application Client Needs: Psychosocial Integrity Integrated Process: Nursing Process/Implementation Content Area: Fundamental Skills References: Jarvis, C. (2004). Physical examination and health assessment (4th ed.). Philadelphia: W.B. Saunders, p. 59. Potter, P., & Perry, A. (2005). Fundamentals of nursing (6th ed.). St. Louis: Mosby, p. 133. 537. A nurse is assigned to care for a client who has just returned to the nursing unit

PN~Comp~Review~CD~501-600~21

following renal biopsy. The nurse plans to do which of the following to properly care for this client for the remainder of the shift? 1. Limit intake of oral fluids 2. Withhold all pain medication 3. Test the urine for occult blood 4. Ambulate the client twice in the hallway Answer: 3 Rationale: Following renal biopsy, serial urine samples are tested for occult blood. The nurse encourages fluid intake to reduce possible clot formation at the biopsy site. Narcotic analgesics are often used to manage the renal colic pain that some clients feel after this procedure. The nurse ensures that the client remains in bed for at least 24 hours. The nurse checks the client’s vital signs and puncture site frequently. Test-Taking Strategy: Use the process of elimination. Begin to answer this question by recalling that encouraging fluid intake will reduce clotting at the site, whereas ambulation could initiate or enhance bleeding at the biopsy site. Thus, options 1 and 4 are eliminated first. Knowing that pain and bleeding are potential concerns after this procedure guides you to select option 3 over option 2. Review postprocedure care following renal biopsy if you had difficulty with this question. Level of Cognitive Ability: Application Client Needs: Physiological Integrity Integrated Process: Nursing Process/Planning Content Area: Adult Health/Renal Reference: Pagana, K., & Pagana, T. (2003). Mosby’s diagnostic and laboratory test reference (6th ed.). St. Louis: Mosby, pp. 756-757. 538. A nurse is assisting a client with cystitis to select foods that are appropriate for an acid-ash diet. The nurse encourages the client to eat which of the following foods? 1. Ice cream 2. Cheese 3. Garden peas 4. Strawberries Answer: 2 Rationale: Foods that are allowed on an acid-ash diet include meat, fish, shellfish, cheese, eggs, poultry, grains, cranberries, prunes, plums, corn, lentils, and foods with high amounts of chlorine, phosphorus, and sulfur. Foods that are not included are milk and milk products such as ice cream, all vegetables except corn and lentils, all fruits except cranberries, plums, and prunes, and foods containing high amounts of sodium, potassium, calcium, and magnesium. Test-Taking Strategy: Focus on the issue—an acid-ash diet. Recalling the foods allowed in this diet will direct you to option 2. Review this diet if you had difficulty with this question. Level of Cognitive Ability: Application Client Needs: Physiological Integrity Integrated Process: Nursing Process/Implementation Content Area: Adult Health/Renal References: deWit, S. (2005). Fundamental concepts and skills for nursing. Philadelphia:

PN~Comp~Review~CD~501-600~22

W.B. Saunders, p. 536. Nix, S. (2005). Williams’ basic nutrition & diet therapy (11th ed.). St. Louis: Mosby, pp. 406-407. 539. A nurse is assigned to care for a client who has just returned to the nursing unit after having hemodialysis for the first time. The nurse monitors the client carefully for which signs and symptoms of disequilibrium syndrome? 1. Vomiting and headaches 2. Abdominal pain and hypotension 3. Lethargy and hypertension 4. Hypertension and sleepiness Answer: 1 Rationale: A complication that can occur during early dialysis is disequilibrium syndrome. This syndrome results from a high osmotic gradient in the brain following the rapid fluid removal that can occur during hemodialysis. Because solutes are not removed as quickly from the cerebrospinal fluid (CSF) and brain, fluid from the circulation shifts into the brain, causing cerebral edema. The client may exhibit nausea and vomiting, confusion, headaches, restlessness, twitching, muscle cramps, and seizures. Options 2, 3, and 4 do not identify signs of disequilibrium syndrome. Test-Taking Strategy: Use the process of elimination. Eliminate options 3 and 4 first because they are similar. From the remaining options, focusing on the name of the syndrome will direct you to option 1. Review the signs and symptoms of disequilibrium syndrome if you had difficulty with this question. Level of Cognitive Ability: Application Client Needs: Physiological Integrity Integrated Process: Nursing Process/Data Collection Content Area: Adult Health/Renal Reference: Linton, A., & Maebius, N. (2003). Introduction to medical-surgical nursing (3rd ed.). Philadelphia: W.B. Saunders, p. 784. 540. A nurse is caring for a woman in labor. The nurse is monitoring the baseline fetal heart rate (FHR) and would document that the FHR is normal if which of the following were noted? 1. 105 beats per minute (BPM) 2. 150 BPM 3. 170 BPM 4. 180 BPM Answer: 2 Rationale: The normal baseline FHR has a lower limit of 120 BPM and an upper limit of 150 to 160 BPM. Baseline bradycardia is a FHR less than 120 BPM. Mild bradycardia is 110 to 119 BPM. Severe bradycardia is less than 100 BPM. Baseline tachycardia is a FHR above 160 BPM. Mild tachycardia is 160 to 180 BPM, and severe tachycardia is greater than 180 BPM. Test-Taking Strategy: Use the process of elimination. Eliminate options 3 and 4 because they are excessively high rates. Next eliminate option 1 because it is a very low FHR. Review the normal baseline FHR if you had difficulty with this question.

PN~Comp~Review~CD~501-600~23

Level of Cognitive Ability: Comprehension Client Needs: Physiological Integrity Integrated Process: Communication and Documentation Content Area: Maternity/Intrapartum Reference: Leifer, G. (2005). Maternity nursing (9th ed.). Philadelphia: W.B. Saunders, p. 75. 541. A nurse is reinforcing the discharge plan with a female teenager with anorexia nervosa. The nurse reinforces the importance of the teenager attending a meeting of the local chapter of Anorexia Nervosa and Associated Disorders. Which response by the teenager indicates that she will most likely be compliant with this plan? 1. “I’ll go once but if I don’t like it I won’t go back.” 2. “I’ll think about it.” 3. “I’ll do whatever I have to do to get out of this place.” 4. “I’m going to do whatever it takes to get better.” Answer: 4 Rationale: Self-help groups serve to reduce the possibilities of further emotional distress leading to pathology and necessary treatment. Option 1 indicates that the client already has doubts about participation and has given herself permission to terminate prior to giving it an initial try. Option 2 displays an ambivalent attitude that promises nothing. Option 3 indicates that the client’s thinking is limited to short-term goals. Option 4 shows that the client is a proactive participant in her plan of care. Test-Taking Strategy: Note the key words most likely in the question. Use the process of elimination, selecting the option that demonstrates the most positive client response in terms of participation. Review the purpose of support groups and the indicators of client compliance with a prescribed treatment plan if you had difficulty with this question. Level of Cognitive Ability: Analysis Client Needs: Psychosocial Integrity Integrated Process: Nursing Process/Evaluation Content Area: Mental Health Reference: Morrison-Valfre, M. (2005). Foundations of mental health care (3rd ed.). St. Louis: Mosby, p. 240. 542. A nurse is caring for a 3-hour-old infant and notes that the infant has not eaten since birth, is jittery, and has a weak cry. The mother states that she cannot get the baby to eat. What action should the nurse take first? 1. Let the infant sleep 2. Check the blood glucose level 3. Ask the registered nurse to call the physician immediately 4. Feed the infant Answer: 2 Rationale: This infant has classic symptoms of hypoglycemia. The nurse should plan to check the infant’s blood glucose level to determine the extent of hypoglycemia, if any, and then to take action by calling the physician or feeding the infant as per the policy of the agency. Permitting the infant to sleep may cause the hypoglycemia to remain untreated and result in neurological damage.

PN~Comp~Review~CD~501-600~24

Test-Taking Strategy: Use the process of elimination and note the key word first. Focus on the data in the question to determine that the infant may be experiencing hypoglycemia. This will direct you to option 2. Also note that this is the only option that addresses data collection, the first step in the nursing process. Review the signs of hypoglycemia in the infant and appropriate nursing interventions if you had difficulty with this question. Level of Cognitive Ability: Application Client Needs: Physiological Integrity Integrated Process: Nursing Process/Implementation Content Area: Maternity/Postpartum Reference: Leifer, G. (2005). Maternity nursing (9th ed.). Philadelphia: W.B. Saunders, p. 278. 543. A nurse teaches a client how to use an incentive spirometer. Which observation would indicate ineffective use of the spirometer by the client? 1. The client inhales slowly 2. The client forms a tight seal around the mouthpiece with the lips 3. The client is breathing through the nose 4. The client removes the mouthpiece from the mouth to exhale Answer: 3 Rationale: Incentive spirometry is not effective if the client breathes through the nose. The client should exhale, form a tight seal around the mouthpiece, inhale slowly, hold to the count of three, and remove the mouthpiece to exhale. The client should repeat the exercise approximately 10 times every hour for best results. Test-Taking Strategy: Use the process of elimination and note the words “ineffective use.” Visualizing the use of the incentive spirometer will assist in directing you to the correct option. Review this procedure if you had difficulty with this question. Level of Cognitive Ability: Comprehension Client Needs: Health Promotion and Maintenance Integrated Process: Nursing Process/Evaluation Content Area: Adult Health/Respiratory Reference: deWit, S. (2005). Fundamental concepts and skills for nursing. Philadelphia: W.B. Saunders, pp. 516-517. 544. What equipment should the nurse plan to have at the bedside when initiating a clear liquid diet in a postoperative client who has had general anesthesia? 1. Oxygen via nasal cannula 2. Suction equipment 3. Cardiac monitor 4. A straw Answer: 2 Rationale: General anesthesia depresses the gag reflex, which in turn increases the risk for aspiration. Suction equipment must be available in the event that the client aspirates. Oxygen may be administered postoperatively and a cardiac monitor may be present, but they have nothing to do with initiation of postoperative diet intake. A straw may help the client sip fluids, but is not necessary. Test-Taking Strategy: Focus on the issue of the question, which is the risk for aspiration

PN~Comp~Review~CD~501-600~25

and airway clearance. This will direct you to option 2. Option 2 maintains airway clearance. Review care to the postoperative client if you had difficulty with this question. Level of Cognitive Ability: Application Client Needs: Physiological Integrity Integrated Process: Nursing Process/Planning Content Area: Fundamental Skills Reference: Christensen, B., & Kockrow, E. (2003). Adult health nursing (4th ed.). St. Louis: Mosby, p. 41. 545. A nurse is reinforcing discharge instructions to a client who has had a total gastrectomy. The nurse tells the client about the importance of returning to the health care clinic as scheduled for which priority assessment? 1. Vitamin B12 and folic acid studies 2. Renal function studies 3. Vital sign measurements 4. Gastric analysis studies Answer: 1 Rationale: Common nutritional problems following stomach removal include vitamin B12 and folic acid deficiency. This may result from a deficiency of an intrinsic factor and/or inadequate absorption because food enters the bowel too quickly. Option 3 may be a component of the assessment at a follow-up health care visit but is not a priority assessment. Options 2 and 4 are not necessary studies following a total gastrectomy. Test-Taking Strategy: Focus on the issue—the complications associated with gastrectomy. Recalling that vitamin B12 deficiency occurs with this type of surgery will direct you to option 1. Review the complications following gastrectomy if you had difficulty with this question. Level of Cognitive Ability: Application Client Needs: Physiological Integrity Integrated Process: Nursing Process/Implementation Content Area: Adult Health/Gastrointestinal Reference: Linton, A., & Maebius, N. (2003). Introduction to medical-surgical nursing (3rd ed.). Philadelphia: W.B. Saunders, p. 690. 546. A client is seen in the health care clinic, and acute pyelonephritis is suspected. The nurse reviews the client’s record and would expect to note which associated signs and symptoms documented? 1. Nausea and vomiting 2. Flank pain on the unaffected side 3. Low-grade fever 4. Pale, dilute urine Answer: 1 Rationale: Typical manifestations of acute pyelonephritis include high fever, chills, nausea, vomiting, and flank pain on the affected side with costovertebral angle tenderness, general weakness, and headache. The client often exhibits the typical signs and symptoms of cystitis, with production of urine that is foul-smelling and cloudy or bloody and that has an increased white blood cell (WBC) count.

PN~Comp~Review~CD~501-600~26

Test-Taking Strategy: Use the process of elimination. The least likely option is flank pain on the unaffected side, so option 2 is eliminated first. Pale, dilute urine is the next unlikely option, since infection usually causes the urine to become bloody or at least turbid. To select between the remaining options, you need to know that pyelonephritis causes high fever, chills, nausea, and vomiting, which would direct you to option 1. Review the manifestations of acute pyelonephritis if you had difficulty with this question. Level of Cognitive Ability: Comprehension Client Needs: Physiological Integrity Integrated Process: Nursing Process/Data Collection Content Area: Adult Health/Renal Reference: Linton, A., & Maebius, N. (2003). Introduction to medical-surgical nursing (3rd ed.). Philadelphia: W.B. Saunders, p. 769. 547. A nurse is reinforcing discharge instructions to a client following surgical treatment for carpal tunnel syndrome. Which statement by the client would indicate a need for further instruction? 1. “I should elevate my arm to reduce the swelling.” 2. “I should use a sling to limit movement and keep my arm elevated.” 3. “I should return to the physician in about 10 days to have the sutures removed.” 4. “I should perform pronation and supination exercises of my wrist starting 24 hours after surgery.” Answer: 4 Rationale: Postoperatively, the client will have a bulky dressing in place for 4 to 7 days. The affected arm is elevated to reduce swelling. A sling is useful to limit movements and to keep the arm elevated. The sutures are removed in about 10 days after surgery. Within 2 to 3 weeks postoperatively, the client will begin physical therapy, with exercises done to promote full range of motion of the wrist and prevent adhesion formation in the carpal tunnel. Test-Taking Strategy: Note the key words need for further instruction. These words indicate a false response question and that you need to select the incorrect client statement. Eliminate options 1 and 2 first because they are similar. Noting the anatomical location of the surgery will assist you in eliminating option 3 and direct you to option 4 as the correct answer to this question. Review postoperative teaching points for the client with carpal tunnel syndrome if you had difficulty with this question. Level of Cognitive Ability: Analysis Client Needs: Physiological Integrity Integrated Process: Teaching/Learning Content Area: Adult Health/Musculoskeletal Reference: Linton, A., & Maebius, N. (2003). Introduction to medical-surgical nursing (3rd ed.). Philadelphia: W.B. Saunders, p. 818. 548. A nursing student is caring for a hospitalized client with a diagnosis of lung cancer. The physician has ordered a partial rebreathing facemask for the client, and the nursing instructor asks the student about the purpose of the partial rebreather facemask. The student correctly responds by stating that: 1. “The mask requires a low liter flow to prevent rebreathing of carbon dioxide.”

PN~Comp~Review~CD~501-600~27

2. “The device conserves oxygen by having the client rebreathe his or her own exhaled air.” 3. “The mask requires that the reservoir bag be deflated to work effectively.” 4. “The device delivers accurate fraction of inspired oxygen (FiO2) to the client.” Answer: 2 Rationale: Rebreathing masks have a reservoir bag that conserves oxygen and requires a high liter flow to achieve concentrations of 40% to 60%. It does not deliver accurate FiO2 to the client. The bag should not deflate during inspiration. The rebreathing bags conserve oxygen by having the client rebreathe his or her own exhaled air. Test-Taking Strategy: Use the process of elimination. Note the relationship between the words “partial rebreather” in the question and “rebreathe his or her own exhaled air” in the correct option. Review oxygen delivery systems if you had difficulty with this question. Level of Cognitive Ability: Comprehension Client Needs: Physiological Integrity Integrated Process: Teaching/Learning Content Area: Adult Health/Respiratory Reference: deWit, S. (2005). Fundamental concepts and skills for nursing. Philadelphia: W.B. Saunders, pp. 507-508. 549. A nurse is caring for a client following total hip replacement who has a Hemovac wound suction in place. At the end of the 8-hour shift, the nurse empties 45 mL of drainage from the wound suction device. Based on this amount of drainage, which action is appropriate? 1. Document the findings 2. Check the client’s blood pressure 3. Notify the registered nurse immediately 4. Place the leg in a flat position Answer: 1 Rationale: Following total hip replacement, the hip incision may have a wound suction drain in place, which is expected to drain usually less than 50 mL every 8 hours. The nurse would document the findings. The nurse may check the client’s blood pressure, but this action is unrelated to the amount of drainage from the Hemovac. There is no need to notify the registered nurse immediately. Placing the leg flat in bed would be done only if prescribed by the physician. Additionally, this action is unrelated to the issue of the question. Test-Taking Strategy: Focus on the issue of the question and the amount of drainage noted. Recalling the normal postoperative findings following total hip replacement will direct you to option 1. Review these postoperative expectations if you had difficulty with this question. Level of Cognitive Ability: Application Client Needs: Physiological Integrity Integrated Process: Nursing Process/Implementation Content Area: Adult Health/Musculoskeletal References: Lewis, S., Heitkemper, M., & Dirksen, S. (2004). Medical-surgical nursing: Assessment and management of clinical problems (6th ed.). St. Louis: Mosby, p. 408.

PN~Comp~Review~CD~501-600~28

Phipps, W., Monahan, F., Sands, J., Marek, J., & Neighbors, M. (2003). Medical-surgical nursing: Health and illness perspectives (7th ed.). St. Louis: Mosby, p. 1535. 550. A nurse is reinforcing instructions to the family of a client with Alzheimer’s disease regarding tacrine (Cognex). Which statement by the family would indicate an understanding of the adverse effects related to this medication? 1. “I need to call the physician if constipation occurs.” 2. “Fever is a sign of an adverse effect of the medication.” 3. “Increased urination may be an indication of an adverse effect.” 4. “If difficulty voiding occurs, I need to call the physician immediately because the medication will need to be discontinued.” Answer: 3 Rationale: Tacrine is a cholinergic agent. Frequent side effects of this medication include nausea, vomiting, diarrhea, dizziness, and headache. Overdose (adverse effects) cause cholinergic crisis, including increased salivation, lacrimation, urination, defecation, bradycardia, hypotension, and increased muscle weakness. Test-Taking Strategy: Use the process of elimination. If you can remember that this medication is a cholinergic agent, it will assist you in determining that an overdose will cause cholinergic symptoms. This knowledge will direct you to option 3. Review cholinergic symptoms and this medication if you had difficulty with this question. Level of Cognitive Ability: Analysis Client Needs: Physiological Integrity Integrated Process: Nursing Process/Evaluation Content Area: Pharmacology Reference: Hodgson, B., & Kizior, R. (2005). Saunders nursing drug handbook 2005. Philadelphia: W.B. Saunders, p. 1001. 551. A nurse is caring for a client with a diagnosis of metastatic breast carcinoma. Tamoxifen citrate (Nolvadex) 10 mg orally twice daily is prescribed for the client, and the nurse provides instructions to the client regarding the medication. Which statement by the client would indicate an understanding of the medication? 1. “Pelvic pain and pressure from this medication are expected.” 2. “If I have difficulty seeing, I need to call the physician.” 3. “If hot flashes occur, I need to call the physician.” 4. “Vaginal bleeding should not occur with this medication.” Answer: 2 Rationale: Tamoxifen citrate is an antineoplastic medication that competes with estradiol for binding to estrogen in tissues containing a high concentration of receptors, such as the breasts, the uterus, and the vagina. Frequent side effects include hot flashes, nausea, vomiting, vaginal bleeding or discharge, pruritis vulvae, and skin rash. Adverse or toxic reactions include retinopathy, corneal opacity, and decreased visual acuity. The client needs to report menstrual irregularities, pelvic pain or pressure, and visual disturbances. Test-Taking Strategy: Knowledge of tamoxifen citrate is necessary to answer this question. Reading the options carefully and discriminating side effects from adverse effects will direct you to option 2. Review the teaching points regarding this medication if you had difficulty with this question.

PN~Comp~Review~CD~501-600~29

Level of Cognitive Ability: Analysis Client Needs: Physiological Integrity Integrated Process: Nursing Process/Evaluation Content Area: Pharmacology Reference: Skidmore-Roth, L. (2005). Mosby’s drug guide for nurses (6th ed.). St. Louis: Mosby, p. 815. 552. A nurse is assigned to care for a client admitted to the postpartum unit following delivery of a full-term healthy infant. The nurse checks the mother’s temperature and notes that it is 100.4° F (38° C). Which nursing action would be appropriate? 1. Encourage oral fluids 2. Notify the registered nurse immediately 3. Administer acetaminophen (Tylenol) 4. Remove the blankets from the mother and recheck her temperature in 30 minutes Answer: 1 Rationale: Temperatures up to 100.4° F (38° C) in a mother during the first 24 hours after birth are often related to the dehydrating effects of labor. Increasing hydration by encouraging oral fluids will help bring the temperature to a normal reading. Options 2, 3, and 4 are unnecessary actions at this time. Test-Taking Strategy: Focus on the data in the question and recall the normal findings in the mother in the immediate postpartum period. Knowing that labor has a dehydrating effect will direct you to option 1. Review the expected postpartum findings if you had difficulty with this question. Level of Cognitive Ability: Application Client Needs: Physiological Integrity Integrated Process: Nursing Process/Implementation Content Area: Maternity/Postpartum Reference: Leifer, G. (2005). Maternity nursing (9th ed.). Philadelphia: W.B. Saunders, pp. 196-197. 553. A nurse in the postpartum unit is assigned to care for a client who delivered a fullterm healthy baby. The nurse receives report and is told that the mother had lost 500 mL of blood during the delivery. When checking the vital signs, the nurse notes that the woman’s pulse is 90 beats per minute (BPM) and is weak and thready. This finding would indicate which of the following to the nurse? 1. A normal pulse rate following delivery 2. A normal pulse rate following a loss of 500 mL of blood 3. The mother is very excited about the delivery of the baby 4. This may be a sign of hemorrhage or shock Answer: 4 Rationale: A pulse range of 50 to 70 BPM is normal in a mother following delivery and may occur for the first 1 to 2 days after delivery. A weak and thready or rapid pulse is abnormal and may be a sign of hemorrhage or shock. Particular attention should be paid to the pulse rate when there has been a blood loss of 500 mL or greater during or after delivery. Options 1, 2, and 3 are incorrect interpretations. Test-Taking Strategy: Use the process of elimination. Eliminate options 1 and 2 first

PN~Comp~Review~CD~501-600~30

because they are similar. Next note the key words weak and thready. These words should direct you to option 4 because it indicates an abnormality. Review normal postpartum findings if you had difficulty with this question. Level of Cognitive Ability: Analysis Client Needs: Physiological Integrity Integrated Process: Nursing Process/Evaluation Content Area: Maternity/Postpartum Reference: Leifer, G. (2005). Maternity nursing (9th ed.). Philadelphia: W.B. Saunders, p. 197. 554. A nurse is reinforcing instructions to a client who had an episiotomy during the birthing process. Which statement by the client would indicate a need for further instructions? 1. “I can apply a local anesthetic spray to the area.” 2. “I can apply ice to the area.” 3. “I should tighten the perineum before I sit and then relax it slowly after being seated.” 4. “I should take sitz baths three or four times a day and test the water temperature to be sure that it is at 115° F.” Answer: 4 Rationale: The client should be instructed in measures to decrease discomfort following episiotomy and perineal swelling. Gluteal muscle tightening reduces direct pressure on the perineum, so discomfort is minimized. Ice decreases circulation, promotes vasoconstriction, reduces edema, and promotes a local anesthetic effect. Local anesthetic sprays reduce uncomfortable sensations. Heat from sitz baths increases circulation to the perineum, thereby promoting oxygenation and healing, which reduces discomfort. However, the water temperature should not be any greater than 100° F to 105° F. Test-Taking Strategy: Note the key words a need for further instructions. These words indicate a false response question and that you need to select the incorrect client statement. Basic principles related to the temperature of the water for bathing or soaks will direct you to option 4. Review these basic principles if you had difficulty with this question. Level of Cognitive Ability: Comprehension Client Needs: Physiological Integrity Integrated Process: Teaching/Learning Content Area: Maternity/Postpartum Reference: Leifer, G. (2005). Maternity nursing (9th ed.). Philadelphia: W.B. Saunders, p. 246. 555. A nurse is reinforcing instructions to a mother who is bottle-feeding a baby and who is complaining of breast engorgement. Which statement by the client indicates a need for further instructions? 1. “I should avoid wearing a bra at this time.” 2. “I should apply ice packs to the breasts.” 3. “I can take analgesics for the discomfort.” 4. “I should wear a breast binder while my breasts are engorged.” Answer: 1 Rationale: Wearing a bra or applying a breast binder applies pressure, which reduces

PN~Comp~Review~CD~501-600~31

congestion and discomfort. Ice packs reduce circulation and thus congestion and also provide an anesthetic effect. Analgesics help relieve the pain. Test-Taking Strategy: Note the key words need for further instructions. These words indicate a false response question and that you need to select the incorrect client statement. Recalling that breast engorgement involves congestion and discomfort will direct you to option 1. Options 2, 3, and 4 will assist in relieving the discomfort and congestion. Review the measures that will relieve breast engorgement if you had difficulty with this question. Level of Cognitive Ability: Comprehension Client Needs: Physiological Integrity Integrated Process: Teaching/Learning Content Area: Maternity/Postpartum Reference: Lowdermilk, D., & Perry, A. (2004). Maternity & women’s health care (8th ed.). St. Louis: Mosby, pp. 777-778. 556. A nurse assigned to care for a client with cirrhosis reviews the nursing care plan and notes a nursing diagnosis of ineffective breathing pattern related to pressure on the diaphragm. The nurse plans care knowing that which client position will best assist in facilitating breathing? 1. Supine 2. Sims 3. Semi-Fowler’s 4. Prone Answer: 3 Rationale: The client with an ineffective breathing pattern related to pressure on the diaphragm should be placed in a semi-Fowler’s or Fowler’s position. The nurse should support the client’s arms and chest with pillows to facilitate breathing by relieving pressure on the diaphragm. The supine, Sims, and prone positions all are flat positions and would further affect the breathing pattern in the client. Test-Taking Strategy: Use the process of elimination and note the similarities between options 1, 2, and 4. All of these positions are flat positions. Option 3 is the option that is different. Review care to the client with cirrhosis if you had difficulty with this question. Level of Cognitive Ability: Application Client Needs: Physiological Integrity Integrated Process: Nursing Process/Planning Content Area: Adult Health/Gastrointestinal Reference: Linton, A., & Maebius, N. (2003). Introduction to medical-surgical nursing (3rd ed.). Philadelphia: W.B. Saunders, p. 730. 557. A nurse is assisting in checking for Tinel’s sign in a client suspected of having carpal tunnel syndrome (CTS). Which technique would the nurse expect to be used to elicit this sign? 1. Ask the client to flex the wrist at a 90-degree angle for 1 minute and assess for numbness 2. Percuss the medial nerve at the wrist as it enters the carpal tunnel and assess for tingling sensations

PN~Comp~Review~CD~501-600~32

3. Dorsiflex the client’s hand and assess for pain in the forearm 4. Place pressure on the client’s radial nerve and assess for circulation Answer: 2 Rationale: Tinel’s sign is assessed by percussing the medial nerve at the wrist as it enters the carpal tunnel. A tingling sensation over the distribution of the nerve occurs in CTS. Phalen’s maneuver is performed by asking the client to flex the wrist at a 90-degree angle for 1 minute. Numbness and tingling over the distribution of the median nerve, the palmar surface of the thumb, and the index and middle fingers suggest CTS. Phalen’s maneuver is also an indication of CTS. Options 3 and 4 are incorrect. Test-Taking Strategy: Use the process of elimination and knowledge regarding the assessment techniques related to CTS. Note that the words “carpal tunnel” appear both in the question and in the correct option. Review the procedure for checking Tinel’s sign if you had difficulty with this question. Level of Cognitive Ability: Application Client Needs: Health Promotion and Maintenance Integrated Process: Nursing Process/Data Collection Content Area: Adult Health/Neurological Reference: Phipps, W., Monahan, F., Sands, J., Marek, J., & Neighbors, M. (2003). Medical-surgical nursing: Health and illness perspectives (7th ed.). St. Louis: Mosby, p. 1461. 558. A nurse is caring for a client following an abdominal hysterectomy performed 1 day ago. An intravenous (IV) line is infusing and a nasogastric (NG) tube is in place and attached to low intermittent suction. The nurse monitors the client and notes that the bowel sounds are absent. The nurse would appropriately: 1. Notify the registered nurse immediately 2. Remove the NG tube 3. Feed the client 4. Document the finding and continue to check for bowel sounds Answer: 4 Rationale: Bowel sounds may be absent for 3 to 4 postoperative days owing to bowel manipulation during surgery. The nurse should document the finding and continue to monitor the client. The NG tube is not removed and the client should receive nothing by mouth (NPO) until after the onset of bowel sounds. There is no need to notify the registered nurse immediately. Test-Taking Strategy: Use the process of elimination and note the key words 1 day ago. Recalling that bowel sounds may not return for 3 to 4 postoperative days will direct you to option 4. If you had difficulty with this question, review normal postoperative findings. Level of Cognitive Ability: Application Client Needs: Physiological Integrity Integrated Process: Nursing Process/Implementation Content Area: Fundamental Skills References: Christensen, B., & Kockrow, E. (2003). Adult health nursing (4th ed.). St. Louis: Mosby, pp. 47-48. deWit, S. (2005). Fundamental concepts and skills for nursing. Philadelphia: W.B.

PN~Comp~Review~CD~501-600~33

Saunders, p. 749. 559. A nurse is assisting in preparing a teaching plan of care for a client being discharged from the hospital following surgery for testicular cancer. Which instruction will the nurse suggest to include in the plan? 1. An elevation in temperature should be reported to the physician 2. You can lift heavy objects (those weighing 20 pounds or more) after 1 week following surgery 3. You can be fitted for a prosthesis in 6 months 4. You can climb stairs after 1 week Answer: 1 Rationale: For the client who had testicular surgery, the nurse should emphasize the importance of notifying the physician if chills, fever, drainage, redness, or discharge occurs. These symptoms may indicate the presence of an infection. Often a prosthesis is inserted during surgery, so the client does not have to wait 6 months. The nurse instructs the client that he will be able to resume most of his usual activities within 1 week after discharge, except for lifting heavy objects (those weighing 20 pounds or more) and stairclimbing. Test-Taking Strategy: Use the process of elimination. Remember, after any surgical procedure, elevation of temperature could signal an infection and should be reported. Review post-testicular surgery teaching points if you had difficulty with this question. Level of Cognitive Ability: Application Client Needs: Health Promotion and Maintenance Integrated Process: Teaching/Learning Content Area: Adult Health/Oncology Reference: Linton, A., & Maebius, N. (2003). Introduction to medical-surgical nursing (3rd ed.). Philadelphia: W.B. Saunders, p. 991. 560. A nurse is observing a client with chronic obstructive pulmonary disease (COPD) performing the pursed-lip breathing technique. Which observation by the nurse would indicate accurate performance of this breathing technique? 1. The client’s inhalation is twice as long as exhalation 2. The client loosens the abdominal muscles while breathing out 3. The client’s exhalation is twice as long as inhalation 4. The client inhales with pursed lips and exhales with the mouth open wide Answer: 3 Rationale: Prolonging the time for exhaling reduces air trapping due to airway narrowing or collapse in COPD. Tightening the abdominal muscles aids in expelling air. Exhaling through pursed lips increases the intraluminal pressure and prevents the airway from collapsing. Test-Taking Strategy: Use the process of elimination. Recalling that a major purpose of pursed-lip breathing is to prevent air trapping during exhalation will lead you to the correct option. Review the principles of pursed-lip breathing if you are unfamiliar with this technique. Level of Cognitive Ability: Comprehension Client Needs: Physiological Integrity

PN~Comp~Review~CD~501-600~34

Integrated Process: Nursing Process/Evaluation Content Area: Adult Health/Respiratory Reference: Linton, A., & Maebius, N. (2003). Introduction to medical-surgical nursing (3rd ed.). Philadelphia: W.B. Saunders, p. 467. 561. A nurse is caring for a client who is receiving lithium carbonate (lithium) for the treatment of bipolar disorder and monitors the client for signs of lithium toxicity. Which sign would alert the nurse to the potential for toxicity? 1. Constipation 2. Headaches 3. Vomiting 4. Increased urination Answer: 3 Rationale: One of the most common early signs of lithium toxicity is the presence of gastrointestinal (GI) disturbances, such as nausea, vomiting, and diarrhea. Options 1, 2, and 4 are unrelated to lithium toxicity. Test-Taking Strategy: Use the process of elimination. Recalling that GI disturbances are the most common early signs will direct you to option 3. Review the signs of lithium toxicity if you had difficulty with this question. Level of Cognitive Ability: Analysis Client Needs: Physiological Integrity Integrated Process: Nursing Process/Data Collection Content Area: Pharmacology Reference: Lehne, R. (2004). Pharmacology for nursing care (5th ed.). Philadelphia: W.B. Saunders, p. 325. 562. A nurse is assisting in performing a prenatal examination on a client in the third trimester of pregnancy. The primary health care provider performs Leopold’s maneuvers on the client. The nurse understands that the fourth maneuver will assess for which of the following? 1. Fetal lie and presentation 2. Fetal descent 3. Strength of uterine contractions 4. Placenta previa Answer: 2 Rationale: Fetal descent is determined with the fourth maneuver. The first maneuver is to determine the contents of the fundus (either fetal head or breech) and thereby the fetal lie. Leopold’s maneuvers should not be performed during a contraction. Placenta previa is diagnosed by ultrasonography and not by palpation. Test-Taking Strategy: Use the process of elimination. Options 3 and 4 can be eliminated first because they are unrelated to this assessment technique. To select between the remaining options, you must be familiar with these maneuvers. Review Leopold’s maneuvers if you had difficulty with this question. Level of Cognitive Ability: Comprehension Client Needs: Physiological Integrity Integrated Process: Nursing Process/Data Collection

PN~Comp~Review~CD~501-600~35

Content Area: Maternity/Antepartum Reference: Leifer, G. (2005). Maternity nursing (9th ed.). Philadelphia: W.B. Saunders, pp. 106-107. 563. A nurse is monitoring a client with a spinal cord injury who is experiencing spinal shock. Which of the following will provide the nurse with the best information about recovery from the spinal shock? 1. Blood pressure 2. Pulse rate 3. Reflexes 4. Temperature Answer: 3 Rationale: Areflexia characterizes spinal shock; therefore reflexes would provide the best information. Vital sign changes are not consistently affected by spinal shock. Test-Taking Strategy: Use the process of elimination and note the key word best in the question. Recalling the pathophysiology of spinal shock will direct you to option 3. Additionally, note that options 1, 2, and 4 are similar and are vital signs. Review the characteristics of spinal shock if you had difficulty with this question. Level of Cognitive Ability: Analysis Client Needs: Physiological Integrity Integrated Process: Nursing Process/Data Collection Content Area: Adult Health/Neurological Reference: Linton, A., & Maebius, N. (2003). Introduction to medical-surgical nursing (3rd ed.). Philadelphia: W.B. Saunders, p. 439. 564. A nurse is caring for a client with a cerebral aneurysm who is on aneurysm precautions and is monitoring the client for signs of aneurysm rupture. The nurse understands that an early sign of rupture is which of the following? 1. Widened pulse pressure 2. Slowing of pupil response 3. Motor weakness 4. A decline in the level of consciousness Answer: 4 Rationale: Rupture of a cerebral aneurysm usually results in increased intracranial pressure (ICP). The first sign of pressure is a change in the level of consciousness due to compression of the reticular formation in the brain. This change in level of consciousness can be as subtle as drowsiness or restlessness. Because centers that control blood pressure are located lower in the brainstem than those that control consciousness, change in pulse pressure is a later sign. Options 2 and 3 are not early signs of increased ICP. These signs may occur later if the increased ICP has led to neurological damage. Test-Taking Strategy: Note the key word early. Remember that changes in level of consciousness are the first indication of increased ICP. Review the clinical manifestations associated with increased ICP if you had difficulty with this question. Level of Cognitive Ability: Analysis Client Needs: Physiological Integrity Integrated Process: Nursing Process/Data Collection

PN~Comp~Review~CD~501-600~36

Content Area: Adult Health/Neurological Reference: Linton, A., & Maebius, N. (2003). Introduction to medical-surgical nursing (3rd ed.). Philadelphia: W.B. Saunders, pp. 382-384. 565. A client has just returned from the radiology department following an upper gastrointestinal (GI) series. The nurse reviews the physician’s orders expecting to note which of the following needed for routine postprocedure care? 1. Intravenous (IV) fluids 2. A bland diet 3. An antiemetic 4. A laxative Answer: 4 Rationale: Barium sulfate, which is used as contrast material during an upper GI series, is a constipating material. If it is not eliminated from the GI tract, it can cause obstruction. Therefore laxatives or cathartics are administered as part of routine postprocedure care. Options 1, 2, and 3 are not components of postprocedure care following an upper GI series. Test-Taking Strategy: Focus on the diagnostic procedure. Recalling that barium sulfate is administered during this test will direct you to option 4. Review postprocedure care following an upper GI series if you had difficulty with this question. Level of Cognitive Ability: Comprehension Client Needs: Physiological Integrity Integrated Process: Nursing Process/Planning Content Area: Adult Health/Gastrointestinal Reference: Chernecky, C., & Berger, B. (2004). Laboratory tests and diagnostic procedures (4th ed.). Philadelphia: W.B. Saunders, p. 1109. 566. A nurse reviews the nursing care plan developed by a nursing student caring for a client who is receiving continuous tube feedings via a nasogastric (NG) tube. The nurse intervenes if the student documented which intervention in the plan? 1. Check the residual every 4 hours 2. Check for placement before administering medications through the tube 3. Check for placement every 4 hours 4. Keep the feeding bag filled with at least 100 mL of feeding continuously so that it does not run dry Answer: 4 Rationale: The placement of an NG feeding tube is checked at least every 4 hours for residual during administration of continuous tube feedings. Placement is also checked before each bolus with intermittent feedings and before the administration of medications through the tube. The bag and tubing are completely changed every 24 hours or per agency protocol. The hanging bag should be rinsed before new formula is added to it. An excess amount of feeding should not be allowed to sit in the feeding bag because of the potential for the growth of bacteria. Test-Taking Strategy: Note the key word intervenes in the question. This word indicates a false response question and that you need to select the incorrect intervention. Visualize the procedure for continuous tube feedings to assist in answering the question. Eliminate

PN~Comp~Review~CD~501-600~37

options 1, 2, and 3 because they are similar. Review care to the client with a nasogastric feeding tube if you had difficulty with this question. Level of Cognitive Ability: Application Client Needs: Physiological Integrity Integrated Process: Nursing Process/Implementation Content Area: Leadership/Management Reference: deWit, S. (2005). Fundamental concepts and skills for nursing. Philadelphia: W.B. Saunders, p. 477. 567. A nurse is preparing to assist a physician with the insertion of a chest tube. The nurse anticipates that which of the following supplies will be required for the chest tube insertion site? 1. Sterile 4 x 4 gauze pad 2. Sterile Kerlix dressing 3. Petrolatum (Vaseline) gauze 4. Povidone-iodine gauze Answer: 3 Rationale: The first layer of the chest tube dressing is petrolatum (Vaseline) gauze, which allows for an occlusive seal at the chest tube insertion site. Additional layers of gauze cover this layer, and the dressing is secured with a strong adhesive tape or Elastoplast tape. Povidone-iodine may be used to clean the insertion site before insertion of the chest tube. Sterile dressings will be used to cover the Vaseline gauze. Test-Taking Strategy: The key word in this question is required. To answer this question correctly, recall that it is imperative to have an occlusive seal at the site and know which dressing material will provide that seal. Option 3 is the only dressing that will provide that occlusive seal. Review the procedure for inserting a chest tube if you had difficulty with this question. Level of Cognitive Ability: Analysis Client Needs: Physiological Integrity Integrated Process: Nursing Process/Planning Content Area: Adult Health/Respiratory Reference: Perry, A., & Potter, P. (2004). Clinical nursing skills & techniques (5th ed.). St. Louis: Mosby, p. 397. 568. A nurse is caring for a client with a head injury and is monitoring the client for signs of increased intracranial pressure (ICP). Which sign if noted in the client would the nurse report immediately? 1. The client complains of feeling tired 2. The client complains of pain at the site of injury 3. The client vomits 4. The client complains of dizziness when getting out of bed for the first time Answer: 3 Rationale: The client with a closed head injury is at risk of developing increased ICP. This is evidenced by symptoms such as headache, dizziness, confusion, weakness, and vomiting. Options 1 and 2 are expected occurrences. Likewise, option 4 also is an expected occurrence. Option 3 may be an indication of increased ICP, requiring

PN~Comp~Review~CD~501-600~38

notification of the registered nurse and physician. Test-Taking Strategy: Use the process of elimination. Eliminate option 2 first, knowing that pain at the site of injury is likely to occur. Next eliminate option 1 because it is likely that the client may feel tired following the trauma of a head injury. From the remaining options, recall that most clients experience some dizziness when getting out of bed following any traumatic event. This will direct you to option 3. Review the signs of increased ICP if you had difficulty with this question. Level of Cognitive Ability: Analysis Client Needs: Physiological Integrity Integrated Process: Nursing Process/Data Collection Content Area: Adult Health/Neurological Reference: Linton, A., & Maebius, N. (2003). Introduction to medical-surgical nursing (3rd ed.). Philadelphia: W.B. Saunders, pp. 382-383. 569. An emergency room nurse is assigned to assist in caring for a client who has suffered a head injury following a motor vehicle accident. The nurse understands that the initial data collection should focus on which of the following? 1. Respiratory status 2. Apical pulse rate 3. Level of consciousness 4. Range-of-motion status of the lower extremities Answer: 1 Rationale: The initial data collection focuses on ensuring that the client has an adequate airway and respiratory status. In rapid sequence, the client’s circulatory status is evaluated, followed by evaluation of the neurological status. Range of motion is not a priority. In fact, the extent of the injuries should be well established prior to assessing range of motion. Test-Taking Strategy: In emergency situations, remember to use the ABCs—airway, breathing, and circulation. The correct answer will most often be the option that deals with the client’s airway. Respiratory status supports this action. Review initial care to the client following a head injury if you had difficulty with this question. Level of Cognitive Ability: Comprehension Client Needs: Physiological Integrity Integrated Process: Nursing Process/Data Collection Content Area: Adult Health/Neurological Reference: Linton, A., & Maebius, N. (2003). Introduction to medical-surgical nursing (3rd ed.). Philadelphia: W.B. Saunders, p. 194. 570. A nurse is caring for a client with a spinal cord injury. The nurse prepares to place high-top sneakers on the client’s feet to prevent the occurrence of: 1. Plantar flexion 2. Foot drop 3. Pressure ulcers 4. Deep vein thrombosis Answer: 2 Rationale: The most effective way to prevent foot drop is to use posterior splints or high-

PN~Comp~Review~CD~501-600~39

top sneakers. A foot board prevents plantar flexion but also places the client at greater risk for developing pressure ulcers of the feet. Pneumatic boots prevent deep vein thrombosis, but not foot drop. Test-Taking Strategy: Use the process of elimination. Focus on each of the conditions identified in the options and the measures that will prevent each of these conditions. Options 3 and 4 can then be easily eliminated. From the remaining options, visualize high-top sneakers on the client. This should help direct you to option 2. Review the measures that will prevent foot drop if you had difficulty with this question. Level of Cognitive Ability: Application Client Needs: Physiological Integrity Integrated Process: Nursing Process/Implementation Content Area: Adult Health/Neurological Reference: Black, J., & Hawks, J. (2005). Medical-surgical nursing: Clinical management for positive outcomes (7th ed.). Philadelphia: W.B. Saunders, p. 2127. 571. A Halo vest is applied to a client following a cervical spine fracture. The nurse provides instructions to the client regarding safety measures related to the vest. Which statement by the client indicates a need for further instructions? 1. “I will bend at the waist, keeping the Halo vest straight to pick up items.” 2. “I will use a walker for ambulating if I need to.” 3. “I will wear rubber-soled shoes for walking.” 4. “I will scan the room to see things.” Answer: 1 Rationale: The client with a Halo vest should avoid bending at the waist, because the Halo vest is heavy and the client’s trunk is limited in flexibility. It is helpful for the client to scan the environment visually because the client’s peripheral vision is diminished from keeping the neck in a stationary position. Use of a walker and rubber-soled shoes may help prevent falls and injury, so these items are also helpful. Test-Taking Strategy: Use the process of elimination. Note the key words need for further instructions. These words indicate a false response question and that you need to select the action that could put the client at risk for injury. Visualize each of the items or actions in the options to assist in identifying how injury could be prevented. Review client teaching points related to a Halo vest if you had difficulty with this question. Level of Cognitive Ability: Analysis Client Needs: Health Promotion and Maintenance Integrated Process: Teaching/Learning Content Area: Adult Health/Neurological Reference: Phipps, W., Monahan, F., Sands, J., Marek, J., & Neighbors, M. (2003). Medical-surgical nursing: Health and illness perspectives (7th ed.). St. Louis: Mosby, p. 1414. 572. A nurse is preparing a plan of care to monitor for complications in a client who will be returning from the operating room following transsphenoidal resection of a pituitary adenoma. Which of the following does the nurse document in the plan as the priority nursing intervention for this client? 1. Monitor apical pulse rate

PN~Comp~Review~CD~501-600~40

2. Monitor temperature 3. Monitor urine output 4. Monitor blood pressure Answer: 3 Rationale: The most common complication of surgery on the pituitary gland is temporary diabetes insipidus. This results from deficiency in antidiuretic hormone (ADH) secretion as a result of surgical trauma. The nurse measures the client’s urine output to determine whether this complication is occurring. Options 1, 2, and 4 are also components of the plan, but option 3 clearly identifies the priority intervention for this type of surgery. Test-Taking Strategy: To answer this question correctly, you should recall that the pituitary gland is responsible for the production of ADH. This allows you to eliminate each of the incorrect options systematically and directs you to option 3. Additionally, note that options 1, 2, and 4 are similar and relate to vital signs. Review the complications of this surgical procedure if you had difficulty with this question. Level of Cognitive Ability: Analysis Client Needs: Physiological Integrity Integrated Process: Nursing Process/Planning Content Area: Adult Health/Neurological Reference: Christensen, B., & Kockrow, E. (2003). Adult health nursing (4th ed.). St. Louis: Mosby, pp. 458-459. 573. A nurse is completing the laboratory requisition that will accompany an arterial blood gas (ABG) specimen sent to the laboratory for analysis. The nurse understands that which of the following data will not be required by the laboratory for adequate evaluation of the specimen? 1. The date and time the specimen was drawn 2. A list of client allergies 3. Any supplemental oxygen the client is receiving 4. The client’s temperature Answer: 2 Rationale: An ABG requisition usually contains information about the date and time the specimen was drawn, the client’s temperature, whether the specimen was drawn on room air or using supplemental oxygen, and the ventilator settings if the client is on a mechanical ventilator. A list of the client’s allergies is not a necessary piece of information required for analysis of the specimen. Test-Taking Strategy: Note the key word not. This word indicates a false response question and that you need to select the unnecessary data. Review the options from the viewpoint of the relevance of the item to the client’s airway status or oxygen utilization. The client’s allergies do not have a direct bearing on the laboratory results. Review this laboratory test if you had difficulty with this question. Level of Cognitive Ability: Application Client Needs: Physiological Integrity Integrated Process: Nursing Process/Implementation Content Area: Adult Health/Respiratory Reference: Chernecky, C., & Berger, B. (2004). Laboratory tests and diagnostic procedures (4th ed.). Philadelphia: W.B. Saunders, pp. 248-249.

PN~Comp~Review~CD~501-600~41

574. A nurse is reinforcing discharge instructions to a client who had a unilateral adrenalectomy. Which of the following will be a component of the instructions? 1. The need for lifelong replacement of all adrenal hormones 2. Instructions about early signs of a wound infection 3. The reason for maintaining a diabetic diet 4. Instruction regarding the proper application of an ostomy pouch Answer: 2 Rationale: A client who is undergoing a unilateral adrenalectomy will be placed on corticosteroids temporarily to avoid a cortisol deficiency. These medications will be gradually weaned in the postoperative period until they are discontinued. Because of the anti-inflammatory properties of corticosteroids, clients who undergo an adrenalectomy are at increased risk of developing wound infections. Because of this increased risk of infection, it is important for the client to know measures to prevent infection, early signs of infection, and what to do if an infection seems to be present. Options 1, 3, and 4 are incorrect instructions. Test-Taking Strategy: Use the process of elimination. When reading the question, note that only one adrenal gland was removed. Recalling that the hormones from the adrenal glands are needed for proper immune system function will assist in eliminating options 3 and 4. From the remaining options, recognizing that one gland can take over the function of two adrenal glands will direct you to option 2. Review postoperative teaching points following adrenalectomy if you had difficulty with this question. Level of Cognitive Ability: Application Client Needs: Physiological Integrity Integrated Process: Teaching/Learning Content Area: Adult Health/Endocrine Reference: Linton, A., & Maebius, N. (2003). Introduction to medical-surgical nursing (3rd ed.). Philadelphia: W.B. Saunders, p. 877. 575. A nurse is reinforcing discharge instructions to a client who has undergone transsphenoidal surgery for a pituitary adenoma. Which statement by the client indicates that the client understands the discharge instructions? 1. “I need to remove the nasal packing in 1 week.” 2. “I need to cough and deep breathe every 2 hours.” 3. “I can take acetaminophen (Tylenol) if I get a severe headache.” 4. “I need to call the doctor if I develop frequent swallowing or postnasal drip.” Answer: 4 Rationale: The client should report frequent swallowing or postnasal drip after transsphenoidal surgery because it could indicate cerebrospinal fluid (CSF) leakage. The surgeon removes the nasal packing, usually after 24 hours. The client should deep breathe, but coughing is contraindicated because it could cause increased intracranial pressure (ICP) . The client should also report a severe headache because it could indicate increased ICP. Test-Taking Strategy: Noting the anatomical location of this surgery will provide the clue that the concern is increased ICP and CSF leakage. Option 1 can be easily eliminated because the physician, not the client, will remove the packing. Options 2 and 3 can be

PN~Comp~Review~CD~501-600~42

eliminated next. Coughing can cause increased ICP, and a severe headache is an indication of such. Review postoperative teaching following this surgical procedure if you had difficulty with this question. Level of Cognitive Ability: Comprehension Client Needs: Physiological Integrity Integrated Process: Nursing Process/Evaluation Content Area: Adult Health/Neurological Reference: Linton, A., & Maebius, N. (2003). Introduction to medical-surgical nursing (3rd ed.). Philadelphia: W.B. Saunders, p. 861. 576. Acetylsalicylic acid (aspirin) is prescribed for a client before a percutaneous transluminal coronary angioplasty (PTCA). When the nurse brings the aspirin to the client, the client asks the nurse about its purpose. The nurse informs the client that the aspirin will: 1. Prevent a fever after the procedure 2. Relieve pain at the injection site 3. Prevent the formation of clots 4. Prevent inflammation of the injection site Answer: 3 Rationale: Before PTCA, the client is usually given an anticoagulant, commonly aspirin, to help reduce the risk of occlusion of the artery during the procedure. Options 1, 2, and 4 are unrelated to the purpose of administering aspirin to this client. Test-Taking Strategy: Use the process of elimination and recall the action and properties of aspirin. Options 2 and 4 can be eliminated first because they are similar. Awareness of the potential complications of a PTCA and nursing measures to prevent these complications will help direct you to option 3 from the remaining options. If you had difficulty with this question, review the action and the uses of aspirin and the complications associated with PTCA. Level of Cognitive Ability: Application Client Needs: Physiological Integrity Integrated Process: Nursing Process/Implementation Content Area: Adult Health/Cardiovascular Reference: Skidmore-Roth, L. (2005). Mosby’s drug guide for nurses (6th ed.). St. Louis: Mosby, p. 75. 577. A nurse is caring for a client with coronary artery disease and a topical nitrate is prescribed for the client. The nurse reinforces medication instructions and tells the client that acetaminophen (Tylenol) is usually prescribed to be taken before the administration of the topical nitrate because: 1. Headache is a common side effect of nitrates 2. Acetaminophen potentiates the therapeutic effects of nitrates 3. Acetaminophen does not interfere with platelet action as acetylsalicylic acid (aspirin) does 4. Fever may accompany coronary artery disease Answer: 1 Rationale: Headache occurs as a side effect of nitrates. Acetaminophen may be given

PN~Comp~Review~CD~501-600~43

before nitrates to prevent headaches or to minimize the discomfort from the headaches. Option 2 is incorrect. Options 3 and 4 are unrelated to the issue of the question. Test-Taking Strategy: Use the process of elimination. Eliminate option 2 first, because this is an incorrect statement. Although options 3 and 4 are true statements, they do not address the issue of the question. Also, recalling that headache is a common side effect of nitrates will assist in directing you to the correct option. If you had difficulty with this question, review the side effects of nitrates and the purpose of administering acetaminophen before these medications. Level of Cognitive Ability: Application Client Needs: Physiological Integrity Integrated Process: Teaching/Learning Content Area: Adult Health/Cardiovascular Reference: Lehne, R. (2004). Pharmacology for nursing care (5th ed.). Philadelphia: W.B. Saunders, p. 533. 578. A nurse is preparing a heparin (Liquaemin) infusion for a client with a diagnosis of thrombophlebitis. The nurse asks a licensed practical nurse (LPN) to gather the necessary items that will be needed for initiating and monitoring the infusion therapy. Which of the following items is unnecessary? 1. Intravenous (IV) infusion controller 2. IV tubing 3. Protamine sulfate 4. Vitamin K (AquaMEPHYTON) Answer: 4 Rationale: Vitamin K is the antidote for warfarin sodium (Coumadin). Protamine sulfate is the antidote for heparin. IV tubing will be necessary for connection of the IV solution (containing the prescribed heparin dose) to the client’s IV catheter. Heparin is always infused via an IV pump or controller. Test-Taking Strategy: Note the key word unnecessary. Visualize the process of administration of the heparin infusion. Eliminate options 1 and 2, knowing that IV tubing is necessary and the heparin is always infused by an IV pump or controller. From the remaining options, you must know that the antidote for heparin is protamine sulfate. Review this treatment measure if you had difficulty with this question or are unfamiliar with this medication. Level of Cognitive Ability: Application Client Needs: Safe, Effective Care Environment Integrated Process: Nursing Process/Implementation Content Area: Pharmacology Reference: Lehne, R. (2004). Pharmacology for nursing care (5th ed.). Philadelphia: W.B. Saunders, pp. 549-564. 579. A nurse is assisting in developing a plan of care for a client who will be returning to the nursing unit following a cardiac catheterization via the femoral approach. Which nursing intervention will be included in the postprocedure plan of care? 1. Place the client in Fowler’s position 2. Encourage the client to increase fluid intake

PN~Comp~Review~CD~501-600~44

3. Instruct the client to perform range-of-motion exercises of the extremities 4. Hold precatheterization medications for 24 hours following the procedure Answer: 2 Rationale: Immediately following a cardiac catheterization using the femoral approach, the client should not flex or hyperextend the affected leg. Placing the client in Fowler’s position increases the risk of hemorrhage. Fluids are encouraged to assist in removing the contrast medium from the body. Asking the client to move the toes is done to assess motion, which could be impaired if a hematoma or thrombus were developing. Flexion or hyperextension and range-of-motion exercises of the extremity are contraindicated. The precatheterization medications are needed to treat acute and chronic conditions. Test-Taking Strategy: Note the key words femoral approach in the question. Visualizing this procedure performed by this approach and thinking about the associated complications will assist you in eliminating options 1 and 3. It does not make sense or serve any useful purpose to hold medications from the client following this procedure; therefore select option 2 over option 4. Review postcardiac catheterization care if you had difficulty with this question. Level of Cognitive Ability: Application Client Needs: Physiological Integrity Integrated Process: Nursing Process/Planning Content Area: Adult Health/Cardiovascular Reference: Chernecky, C., & Berger, B. (2004). Laboratory tests and diagnostic procedures (4th ed.). Philadelphia: W.B. Saunders, p. 328. 580. A nurse is reinforcing dietary instructions to a client with congestive heart failure (CHF). The nurse determines that the client understands the instructions if the client states that which of the following food items should be avoided? 1. Leafy green vegetables 2. Catsup 3. Cooked cereal 4. Sherbet Answer: 2 Rationale: Catsup is high in sodium. Leafy green vegetables, cooked cereal, and sherbet all are low in sodium. Clients with CHF should monitor sodium intake. Test-Taking Strategy: Recall that the client with CHF should monitor sodium intake. Use the process of elimination, noting that options 1, 3, and 4 are similar in that they are low-sodium foods. Review the foods high in sodium if you had difficulty with this question. Level of Cognitive Ability: Comprehension Client Needs: Health Promotion and Maintenance Integrated Process: Nursing Process/Evaluation Content Area: Adult Health/Cardiovascular Reference: Christensen, B., & Kockrow, E. (2003). Adult health nursing (4th ed.). St. Louis: Mosby, p. 319. 581. A nurse is assisting in developing a plan of care for a client receiving warfarin sodium (Coumadin). The nurse selects which nursing diagnosis listed in the plan as the

PN~Comp~Review~CD~501-600~45

priority in caring for this client? 1. Risk for excess fluid volume 2. Risk for activity intolerance 3. Risk for injury 4. Risk for infection Answer: 3 Rationale: Anticoagulant therapy predisposes the client to injury because of the inhibitory effects of the medication on the body’s normal blood-clotting mechanism. Bruising, bleeding, and hemorrhage may occur in the course of activities of daily living and with other activities. Options 1, 2, and 4 are not specifically related to the care of a client receiving anticoagulants. Test-Taking Strategy: Recalling that anticoagulants present a risk for bleeding will assist in directing you to option 3. Also note that options 1, 2, and 4 are not directly related to this medication. Review the effects of this medication if you had difficulty with this question. Level of Cognitive Ability: Analysis Client Needs: Safe, Effective Care Environment Integrated Process: Nursing Process/Planning Content Area: Pharmacology Reference: Skidmore-Roth, L. (2005). Mosby’s drug guide for nurses (6th ed.). St. Louis: Mosby, pp. 905-906. 582. A client arrives at the emergency room and complains of severe abdominal pain. The initial diagnosis is an acute condition in the abdomen, and an x-ray and an abdominal ultrasonogram are prescribed to be obtained immediately. The nurse prepares the client for these diagnostic tests and reviews the physician’s orders. Which of the following orders would the nurse question if written on the physician’s order form? 1. Insertion of a nasogastric (NG) tube 2. Insertion of an intravenous (IV) line 3. Administration of an analgesic 4. Maintenance of NPO status Answer: 3 Rationale: Until a differential diagnosis is determined and a decision about the need for surgery is made, the nurse would question an order to give an analgesic because it could mask the client’s symptoms. The nurse can expect the client to be placed on NPO status and to have an IV line inserted. Insertion of an NG tube may be helpful to provide decompression of the stomach. Test-Taking Strategy: Use the process of elimination and note that the wording of the question guides you to look for an option that is an incorrect action. Recalling that an abdominal complaint could result in NPO status and insertion of an NG tube, eliminate options 1 and 4 first. From the remaining options, select option 3 over option 2 because an IV line is a standard, accepted intervention to provide fluids to the client who is NPO. Review care to the client with a diagnosis of acute abdominal symptoms if you had difficulty with this question. Level of Cognitive Ability: Analysis Client Needs: Safe, Effective Care Environment

PN~Comp~Review~CD~501-600~46

Integrated Process: Nursing Process/Implementation Content Area: Adult Health/Gastrointestinal References: Chernecky, C., & Berger, B. (2004). Laboratory tests and diagnostic procedures (4th ed.). Philadelphia: W.B. Saunders, p. 122. Pagana, K., & Pagana, T. (2003). Mosby’s diagnostic and laboratory test reference (6th ed.). St. Louis: Mosby, p. 3. 583. Abdominal ultrasonography is prescribed for a woman who is pregnant. The nurse provides information to the client regarding the procedure and makes which statement to the woman? 1. “You will be positioned on your side with your head flat.” 2. “The procedure will take about 1 hour.” 3. “You need to be sure to urinate before the procedure.” 4. “You will be positioned on your back and turned slightly to one side with your head elevated.” Answer: 4 Rationale: The client is positioned on the back with the head and the knees supported by pillows. The client’s head will be elevated, and the client will be turned slightly to one side to prevent supine hypotension. The procedure takes 10 to 30 minutes. A full bladder makes it easier for sound waves to reach the pelvic area, so the client should be instructed to drink 1 to 2 quarts of clear fluid 1 hour before the test. The client should not void until the ultrasound is obtained. Options 1, 2, and 3 are incorrect. Test-Taking Strategy: Use the process of elimination. Note that both options 1 and 4 address positioning of the client during the test. This indicates that one of these options may be correct. From the remaining options, visualize the procedure to direct you to option 4. Review this procedure if you had difficulty with this question. Level of Cognitive Ability: Application Client Needs: Physiological Integrity Integrated Process: Nursing Process/Implementation Content Area: Maternity/Antepartum Reference: McKinney, E., James, S., Murray, S., & Ashwill, J. (2005). Maternal-child nursing (2nd ed.). St. Louis: Elsevier, p. 325. 584. A client is scheduled for an amniocentesis and tells the nurse, “I’m not sure I should have this test done.” Which response by the nurse is appropriate? 1. “Don’t worry. Everything will be fine.” 2. “The doctor has scheduled this test for a reason.” 3. “Why don’t you want to have this test done?” 4. “Tell me what concerns you have.” Answer: 4 Rationale: The nurse needs to gather more data and assist the client in exploring her feelings about the test. Options 1, 2, and 3 are blocks to communication and are nontherapeutic nursing responses. Test-Taking Strategy: Use therapeutic communication techniques to answer this question. Options 1, 2, and 3 are communication blocks. Option 4 is the only option that addresses the client’s feelings. Remember to address the client’s feelings first. Review

PN~Comp~Review~CD~501-600~47

therapeutic communication techniques if you had difficulty with this question. Level of Cognitive Ability: Application Client Needs: Psychosocial Integrity Integrated Process: Communication and Documentation Content Area: Maternity/Antepartum Reference: McKinney, E., James, S., Murray, S., & Ashwill, J. (2005). Maternal-child nursing (2nd ed.). St. Louis: Elsevier, pp. 30-31. 585. A nurse in the prenatal clinic is taking a nutritional history from a pregnant adolescent. Which statement by the client would alert the nurse to a potential concern regarding adequate nutritional intake during the pregnancy? 1. “I need to gain only 10 pounds so that my baby will be small like I am.” 2. “I really like to have a root beer float with vanilla ice cream in the afternoon.” 3. “I don’t like milk but I can drink it if it is in a shake mixed with chocolate.” 4. “I am not crazy about eating vegetables but I will do my best.” Answer: 1 Rationale: Pregnant adolescents are at higher risk for complications than are other pregnant clients. Adolescents are often concerned about their body image. If weight is a major focus, the adolescent is more likely to restrict calories to avoid weight gain. Option 1 is the only response that suggests a possible concern. Options 2 and 3 indicate that the client will consume items that will help increase calcium intake. Option 4 expresses an attempt to consume required vegetables. Test-Taking Strategy: Note the key words potential concern. Think about the developmental stage of the adolescent. Focus on the issue of body image that is so important for the adolescent. Then, by the process of elimination, you should easily be directed to option 1. Review the developmental stage of the adolescent if you had difficulty with this question. Level of Cognitive Ability: Analysis Client Needs: Psychosocial Integrity Integrated Process: Nursing Process/Data Collection Content Area: Maternity/Antepartum Reference: Leifer, G. (2005). Maternity nursing (9th ed.). Philadelphia: W.B. Saunders, p. 318. 586. A nurse is providing a teaching session to a group of adolescent pregnant clients and is discussing the importance of nutrition. The nurse includes which of the following in the discussion? 1. Encouraging the need to avoid eating at local fast-food restaurants 2. Emphasizing the need to eliminate snack foods 3. Encouraging the adolescents to eat when hungry rather than three times a day 4. Describing the appropriate amount of weight gain required during the pregnancy Answer: 4 Rationale: The developmental stage of the adolescent needs to be addressed when the nurse is providing instructions regarding nutrition during pregnancy. The adolescent should not be told to eliminate favorite foods and places to eat. This may cause the adolescent to rebel. Eating only when hungry could lead to a deficit in nutrients. The adolescent is more likely to follow suggestions when the nurse explains why the weight

PN~Comp~Review~CD~501-600~48

gain is important. Test-Taking Strategy: The client in this question is an adolescent. Consider the growth and development of an adolescent when answering this question. Eliminate option 3 first because it is not a physiologically sound suggestion. Next eliminate options 1 and 2 because these interventions will cause the adolescent to rebel. Review the needs of the pregnant adolescent related to nutrition if you had difficulty with this question. Level of Cognitive Ability: Application Client Needs: Health Promotion and Maintenance Integrated Process: Teaching/Learning Content Area: Maternity/Antepartum Reference: Leifer, G. (2005). Maternity nursing (9th ed.). Philadelphia: W.B. Saunders, p. 318. 587. A nurse is preparing a session regarding nutrition for a group of culturally diverse pregnant women. The nurse determines that the priority nursing intervention includes which of the following? 1. Describe the importance of avoiding eating at fast-food restaurants 2. Obtain the weight of each client 3. Determine the socioeconomic status of each client 4. Identify cultural food preferences Answer: 4 Rationale: The priority nursing intervention is to identify the cultural food preferences of each client. This information is needed in order to adequately provide information regarding appropriate nutrition. The socioeconomic status may be an important component, particularly when the nurse is determining whether a client’s financial situation permits the purchase of appropriate food items. A baseline weight also may be important. Encouraging appropriate nutrition and the need to avoid fast-food restaurants is also important. However, an adequate nutritional regimen can be planned only if cultural food preferences are identified. Test-Taking Strategy: Use the process of elimination to assist you in identifying the physiological needs that are directly related to the individual client. This will easily direct you to option 4. Also note the relation of the word “culturally” in the question and “cultural” in the correct option. Review cultural considerations as they relate to nursing interventions if you had difficulty with this question. Level of Cognitive Ability: Comprehension Client Needs: Psychosocial Integrity Integrated Process: Teaching/Learning Content Area: Maternity/Antepartum Reference: Leifer, G. (2005). Maternity nursing (9th ed.). Philadelphia: W.B. Saunders, p. 53. 588. A nurse is providing dietary instructions to a pregnant client with a history of lactose intolerance. The nurse would instruct the client to consume which best food item to ensure an adequate source of calcium in the diet? 1. Dried fruits 2. Spinach

PN~Comp~Review~CD~501-600~49

3. Orange juice 4. Cheese Answer: 1 Rationale: Dairy products are the best source of calcium. Women with lactose intolerance need other sources of calcium because they are not able to consume dairy products. Calcium is present in dark green, leafy vegetables, broccoli, legumes, nuts, and dried fruits. Spinach contains calcium, but it also contains oxalates that decrease calcium availability. Orange juice does not contain significant amounts of calcium unless it is fortified with calcium. Option 4 is a dairy product and cannot be eaten when the client is lactose intolerant. Test-Taking Strategy: Use the process of elimination and note that the client has a history of lactose intolerance. Option 4 can be eliminated first because it is a dairy product. Recalling that orange juice is not a good source of calcium unless fortified with calcium will assist in eliminating option 3. Spinach contains calcium, but it also contains oxalates that decrease calcium absorption, so eliminate option 2. This leaves option 1 as the correct answer. Review lactose intolerance and foods high in calcium if you had difficulty with this question. Level of Cognitive Ability: Application Client Needs: Health Promotion and Maintenance Integrated Process: Teaching/Learning Content Area: Maternity/Antepartum Reference: McKinney, E., James, S., Murray, S., & Ashwill, J. (2005). Maternal-child nursing (2nd ed.). St. Louis: Elsevier, pp. 314-315. 589. A nurse provides instructions to a pregnant client regarding the administration of iron. The nurse determines that the teaching has been effective if the client states that she will take the iron with which of the following food items? 1. Milk 2. Tea 3. Tomato juice 4. Water Answer: 3 Rationale: Foods containing ascorbic acid (vitamin C), such as tomato juice, may increase the absorption of iron. Additionally, the absorption of iron is affected by many substances. Calcium and phosphorus in milk and tannin in tea decrease iron absorption. Water will not act to increase the absorption of the iron. Test-Taking Strategy: Recall that vitamin C increases the absorption of iron. Next, use the process of elimination to direct you to option 3. Review the teaching points related to the administration of iron if you had difficulty with this question. Level of Cognitive Ability: Comprehension Client Needs: Health Promotion and Maintenance Integrated Process: Teaching/Learning Content Area: Pharmacology Reference: Leifer, G. (2003). Introduction to maternity & pediatric nursing (4th ed.). Philadelphia: W.B. Saunders, p. 61.

PN~Comp~Review~CD~501-600~50

590. A client is seen in the health care clinic for complaints of vaginal bleeding and mild abdominal cramping. On further data collection, the nurse notes that the client’s last menstrual period was 10 weeks ago. The client reports that a home pregnancy test was performed and the results were positive. On physical examination, it is noted that the client has a dilated cervix. The nurse understands that the client is at risk for which type of abortion? 1. Inevitable 2. Incomplete 3. Threatened 4. Septic Answer: 1 Rationale: An inevitable abortion is a termination of pregnancy that cannot be prevented. Moderate to severe bleeding with mild cramping and cervical dilation is present. An incomplete abortion presents with heavy bleeding, severe cramping, cervical dilation, and passage of large clots. A threatened abortion presents with slight to moderate bleeding and intermittent cramping, but no dilation. A septic abortion presents with bleeding with odor, cervical dilation, and fever. Cramping may or may not be present. Test-Taking Strategy: Use the process of elimination. Focus on the client’s symptoms and the physical assessment findings to help you answer the question. There are no supporting data in the question that suggest the occurrence of the types of abortions identified in options 2, 3, and 4. Review the different types of abortion if you had difficulty with this question. Level of Cognitive Ability: Comprehension Client Needs: Physiological Integrity Integrated Process: Nursing Process/Data Collection Content Area: Maternity/Antepartum Reference: Leifer, G. (2005). Maternity nursing (9th ed.). Philadelphia: W.B. Saunders, p. 213. 591. A nurse is reviewing the health history of a pregnant client. Which of the following data if noted in the client’s health history would indicate a risk for spontaneous abortion? 1. Age of 45 years 2. Presence of syphilis 3. Inactive genital herpes 4. Diabetes mellitus Answer: 2 Rationale: Maternal infections such as syphilis, toxoplasmosis, and rubella are causes of spontaneous abortion. There is inconclusive evidence that genital herpes is a causative agent in abortion. Maternal age over 40 years and diabetes mellitus are considered high risk factors in a pregnancy, increasing the risk of congenital malformations. Test-Taking Strategy: Focus on the key words risk for spontaneous abortion. Eliminate options 1 and 4 first because these factors are high risk factors in pregnancy but are not directly related to high risk for abortion. From the remaining options, recall that genital herpes is not necessarily associated with spontaneous abortion. This will direct you to option 2. Review the risk factors associated with spontaneous abortion if you had difficulty with this question.

PN~Comp~Review~CD~501-600~51

Level of Cognitive Ability: Comprehension Client Needs: Health Promotion and Maintenance Integrated Process: Nursing Process/Data Collection Content Area: Maternity/Antepartum Reference: McKinney, E., James, S., Murray, S., & Ashwill, J. (2005). Maternal-child nursing (2nd ed.). St. Louis: Elsevier, p. 617. 592. A nurse is providing emotional support to a client who experienced a spontaneous abortion. The nurse can best assist the client by planning care that focuses on which of the following psychosocial issues? 1. Feelings of guilt are often associated with grief 2. The psychological needs of other children in the family and the ability to bear children in the future should be considered 3. Pain and discomfort occur as a result of the abortion 4. Grief and loss are usually resolved within 3 months Answer: 1 Rationale: Nurses must consider the psychological needs of the family experiencing spontaneous abortion. Grief often includes feelings of guilt. The grieving process is individual and may last a year or longer. It is not appropriate to focus on the client’s ability to have other children. The amount of pain and discomfort is important, but this is a physiological concern. Test-Taking Strategy: The issue of the question relates to a psychosocial need. Use the process of elimination to eliminate option 3 because it describes a physiological concern. Option 2 is a nontherapeutic concern and can be eliminated. Knowledge related to the grieving process will direct you to option 1 from the remaining options. Review the psychosocial issues related to abortion if you had difficulty with this question. Level of Cognitive Ability: Application Client Needs: Psychosocial Integrity Integrated Process: Caring Content Area: Maternity/Antepartum Reference: McKinney, E., James, S., Murray, S., & Ashwill, J. (2005). Maternal-child nursing (2nd ed.). St. Louis: Elsevier, pp. 619-620. 593. A nurse is preparing to collect data on a client with a possible diagnosis of ectopic pregnancy. Which of the following would the nurse check first? 1. Weight 2. Abdominal girth measurement 3. Pulse rate 4. Temperature Answer: 3 Rationale: The primary concern when ectopic pregnancy is suspected is the occurrence of bleeding and hypovolemic shock. Option 3 is the only assessment that would provide information related to this occurrence. An elevated pulse rate is an indicator of shock. The nurse should also monitor for decreasing hematocrit levels and pain. Options 1, 2, and 4 do not provide data that would indicate the occurrence of hypovolemic shock. Test-Taking Strategy: Note the key word first in the question. Knowledge regarding the

PN~Comp~Review~CD~501-600~52

complications associated with ectopic pregnancy will direct you to option 3. Additionally, use the ABCs—airway, breathing, and circulation—to help direct you to the correct option. Review care to the client with a possible diagnosis of ectopic pregnancy if you had difficulty with this question. Level of Cognitive Ability: Application Client Needs: Physiological Integrity Integrated Process: Nursing Process/Data Collection Content Area: Maternity/Antepartum Reference: McKinney, E., James, S., Murray, S., & Ashwill, J. (2005). Maternal-child nursing (2nd ed.). St. Louis: Elsevier, pp. 620-621. 594. A maternity nurse is caring for a client who is admitted to the hospital with a diagnosis of gestational diabetes. This is the client’s first pregnancy. Which statement by the client indicates a knowledge deficit regarding gestational diabetes? 1. “I shouldn’t have eaten so many sweets before I became pregnant.” 2. “Well, I guess I will just have to deal with this.” 3. “Oh, well, I guess this isn’t the end of the world.” 4. “I have heard that this type of diabetes is first discovered during pregnancy.” Answer: 1 Rationale: Gestational diabetes is not necessarily caused by eating too many sweets before pregnancy. Options 2 and 3 indicate a common normal response. Option 4 is an accurate statement. Option 1 is the only option that indicates a knowledge deficit. Test-Taking Strategy: Note the key words knowledge deficit in the question. Options 2 and 3 can be eliminated first because these are common client responses and are unrelated to a knowledge deficit. From the remaining options, recalling the causes of gestational diabetes will direct you to option 1. Review gestational diabetes if you had difficulty with this question. Level of Cognitive Ability: Analysis Client Needs: Psychosocial Integrity Integrated Process: Nursing Process/Evaluation Content Area: Maternity/Antepartum Reference: Leifer, G. (2005). Maternity nursing (9th ed.). Philadelphia: W.B. Saunders, p. 229. 595. A nurse is assigned to care for a client being admitted to the mental health unit following a suicide attempt. The client attempted the suicide by lacerating both wrists. The initial nursing action upon admission of the client is to: 1. Check the wound sites 2. Ask the client about the reason that the suicide act was attempted 3. Encourage and assist the client to vent feelings 4. Administer the prescribed antianxiety agent Answer: 1 Rationale: The physiological integrity of the client is always assessed first. Although options 2, 3, and 4 may be appropriate at some point, the initial action would be to assess the wounds. Test-Taking Strategy: The key word in the question is initial. Use Maslow’s Hierarchy

PN~Comp~Review~CD~501-600~53

of Needs theory to prioritize. Physiological needs come first. Option 1 is the only option that addresses a physiological need. Review initial care to the client who attempted suicide if you had difficulty with this question. Level of Cognitive Ability: Application Client Needs: Physiological Integrity Integrated Process: Nursing Process/Implementation Content Area: Delegating/Prioritizing Reference: Stuart, G., & Laraia, M. (2005). Principles & practice of psychiatric nursing (8th ed.). St. Louis: Mosby, p. 375. 596. A nurse is assigned to care for a pregnant client being admitted to the nursing unit. Laboratory and diagnostic studies have confirmed a diagnosis of gestational trophoblastic disease (hydatidiform mole). The nurse collects data on the client and reviews the results of the laboratory and diagnostic studies knowing that which of the following is an unassociated finding with this diagnosis? 1. Elevated levels of human chorionic gonadotropin (HCG) 2. Vaginal bleeding 3. No fetal heart activity 4. Hypotension Answer: 4 Rationale: The most common signs and symptoms of gestational trophoblastic disease include elevated levels of HCG, vaginal bleeding, larger than normal uterus for gestational age, failure to detect fetal heart activity even with sensitive instruments, excessive nausea and vomiting, and early development of pregnancy-induced hypertension. An elevated blood pressure would also be noted. Test-Taking Strategy: Note the key word unassociated in the question. This word indicates a false response question and that you need to select the incorrect finding. Remember, hypertension would occur in this disorder. Review the clinical findings in gestational trophoblastic disease if you had difficulty with this question. Level of Cognitive Ability: Analysis Client Needs: Physiological Integrity Integrated Process: Nursing Process/Data Collection Content Area: Maternity/Antepartum Reference: Leifer, G. (2005). Maternity nursing (9th ed.). Philadelphia: W.B. Saunders, p. 215. 597. A nurse is providing instructions to a pregnant client regarding the need to consume folic acid in the diet. The nurse determines that the client understands the instructions when the client states that it is necessary to include which of the following food items in the diet? 1. Chicken 2. Rice 3. Cheese 4. Green, leafy vegetables Answer: 4 Rationale: Sources of folic acid include green, leafy vegetables, whole grains, fruits,

PN~Comp~Review~CD~501-600~54

liver, dried peas, and beans. The foods listed in options 1, 2, and 3 are not sources of folic acid. Test-Taking Strategy: Use the process of elimination and knowledge regarding the food items that are high in folic acid to answer this question. Cheese is a dairy product and is high in calcium, and rice and chicken are good sources of iron. Green, leafy vegetables (option 4) are the foods high in folic acid. Review foods high in folic acid if you had difficulty with this question. Level of Cognitive Ability: Comprehension Client Needs: Health Promotion/Maintenance Integrated Process: Nursing Process/Evaluation Content Area: Maternity/Antepartum Reference: Leifer, G. (2003). Introduction to maternity & pediatric nursing (4th ed.). Philadelphia: W.B. Saunders, p. 62. 598. A physician aspirates synovial fluid from the knee joint of a client with rheumatoid arthritis. The nurse reviews the laboratory analysis of the specimen and would expect the results to indicate which finding? 1. Cloudy synovial fluid 2. Presence of organisms 3. Bloody synovial fluid 4. Presence of urate crystals Answer: 1 Rationale: Cloudy synovial fluid is diagnostic of rheumatoid arthritis. Organisms present in the synovial fluid are characteristic of a septic joint condition. Bloody synovial fluid is seen with trauma. Urate crystals are found in gout. Test-Taking Strategy: Use the process of elimination. Remember that organisms indicate infection, blood indicates trauma, and urate crystals indicate gout. Review the characteristics of rheumatoid arthritis if you had difficulty with this question. Level of Cognitive Ability: Analysis Client Needs: Physiological Integrity Integrated Process: Nursing Process/Data Collection Content Area: Adult Health/Immune Reference: Christensen, B., & Kockrow, E. (2003). Adult health nursing (4th ed.). St. Louis: Mosby, p. 112. 599. A nurse is collecting admission data on a client with Parkinson’s disease. The nurse asks the client to stand with the feet together and the arms at the side and then to close the eyes. The nurse notes that the client begins to fall when the eyes are closed. Based on this finding, the nurse documents which of the following in the client’s record? 1. Positive Trousseau’s sign 2. Negative Trousseau’s sign 3. Negative Romberg’s test 4. Positive Romberg’s test Answer: 4 Rationale: Romberg’s test checks for cerebellar functioning related to balance. The client stands with the feet together and the arms at the side and then closes the eyes.

PN~Comp~Review~CD~501-600~55

Slight swaying is normal, but loss of balance indicates a problem and a positive Romberg’s test. Trousseau’s sign indicates a calcium imbalance. Test-Taking Strategy: A clue in the question is that the client has Parkinson’s disease because clients with this disorder often have interferences in balance. Remember, a positive Romberg’s test occurs when the client has a balance problem. Relate Romberg’s test to balance. Review these tests if you had difficulty with this question. Level of Cognitive Ability: Application Client Needs: Physiological Integrity Integrated Process: Nursing Process/Data Collection Content Area: Adult Health/Neurological Reference: Christensen, B., & Kockrow, E. (2003). Adult health nursing (4th ed.). St. Louis: Mosby, p. 585. 600. A nurse is caring for a client with a nasogastric (NG) tube and tests the pH of the aspirate to determine correct placement. The nurse notes that the pH is 5 and determines that the: 1. NG tube needs to be pulled back approximately 1 inch 2. NG tube needs to be reinserted 3. pH of the aspirate needs to be rechecked 4. Placement of the NG tube is accurate Answer: 4 Rationale: After the nurse inserts an NG tube into a client, the correct location of the tube must be verified. Testing the pH of the gastric fluid (determining its acidity) further verifies that the tube is in the stomach. The stomach contents are acidic, and a pH of 5 would indicate accurate placement. Options 1, 2, and 3 are incorrect. Test-Taking Strategy: Recalling that the stomach contents are acidic will assist you in determining that an aspirate with a pH of 5 indicates appropriate placement. If you had difficulty with this question, review NG tube insertion and placement techniques. Level of Cognitive Ability: Analysis Client Needs: Physiological Integrity Integrated Process: Nursing Process/Evaluation Content Area: Adult Health/Gastrointestinal Reference: deWit, S. (2005). Fundamental concepts and skills for nursing. Philadelphia: W.B. Saunders, p. 479.

More Documents from "Linda Kuglarz"

Silvestri Chapter 21 Ed#551
October 2019 28
Silvestri101-200
October 2019 50
Silvestri1301-1400
October 2019 51
Silvestri Chapter 45 Ed#569
October 2019 37
Silvestri Chapter 32 Ed#55c
October 2019 41
Silvestri Chapter 07 Ed#543
October 2019 36